User login
Appropriate cancer screening for women with dense breasts
We have been interested in the quiz series focused on breast cancer screening for women with dense breasts presented in
The concerns with breast cancer in particular
Breast cancer is not cervical cancer. It isn’t one disease. It is a multitude of diseases that happen to show up in the breast. Some are relatively slow-growing—the kinds of cancers that lend themselves to screening and to early intervention. But other cancers are rapidly-growing; they show up no matter how often or what modality we use for screening. Our goal should be to find an approach to screening that can diagnose breast cancer at a stage where we can intervene and positively impact breast cancer specific and overall mortality.
Screening guidelines vary
The variety of screening guidelines published by different professional organizations reflect differing assumptions and sets of values related to the early diagnosis and treatment of breast cancer. (For a comprehensive table of current screening guidelines, see https://www.cdc.gov/cancer/breast/pdf/breast-cancer-screening-guidelines-508.pdf.)
ACOG’s approach—to offer screening at age 40 but to begin by at least age 50 and, through shared decision making with the patient, screen every 1 or 2 years—is focused on capturing as many cases as we can identify, while minimizing the harms of false-positives.1 The perspective of the US Preventive Services Task Force (USPSTF) recommendations (to screen every 2 years beginning at age 50) is at the population level, a cost-effective approach that will have the greatest benefit while minimizing harms in the population at large.2 The American Society of Breast Surgeons recommends screening to begin by age 40.3 Like the breast surgeons, radiologists dedicated to breast imaging are focused on an individual rather than a population level. They strive to identify each and every instance of possible cancer, and therefore recommend annual screening beginning at age 40.4 However, with more aggressive screening in average-risk women many cases of ductal carcinoma in situ (DCIS) are identified—a lesion that, if not detected, may not impact the woman’s health during her lifetime—representing what some might call “overdiagnosis.” Yet there may be some instances in which the DCIS might affect an individual woman’s health. Unfortunately, we can’t prospectively distinguish between the first and the second types of cases.
We follow American College of Obstetricians and Gynecologists and US Preventive Services Task Force guidelines in discussing screening (both its hazards and benefits) with our average-risk patients beginning at age 40. We talk about risk factors for breast cancer, including breast density, but let patients know that no specific additional imaging is advised, and that density is more common in younger women (one consideration in earlier screening) and is quite common in general. Although we do not send follow-up letters to patients with dense breasts, we do educate our staff so that they can respond appropriately should patients call with questions.
Of course, we all bring to the table values that will impact the decisions that we make for ourselves and for our patients. What an ObGyn might suggest may differ from what a radiologist might suggest. Although we follow recommendations made by the radiologist at screening, an ObGyn wants to take care of the whole human being. We are concerned with bones, heart, everything about the patient, so we approach a patient in a different way. These priorities are reflected in the current varying breast cancer screening guidelines.
Continue to: Research on breast cancer screening varies by design...
Research on breast cancer screening varies by design
There has not been a randomized clinical trial conducted on screening mammography since the days of the analog mammogram. The research that has been conducted is difficult to compare due to variations in screening ages and intervals, technology sensitivity, and patient adherence with recommended screening. Treatments for breast cancer also have changed dramatically over time, so the findings of older studies may no longer be relevant to current breast cancer screening. The kind of analysis that needs to be done is an interrupted time series, where you can look at the trajectory of breast cancer survival and whether screening mammography shifts that survival in any way.
One specific study from Australia measured the impact of newer available breast cancer treatments, including tamoxifen for women with receptor-positive tumors and newer chemotherapy strategies.5 The authors analyzed screening mammography trends in one large province where women aged 50 to 69 were offered biennial screening. Trends from the 1990s showed that more women were being screened over time. Simultaneously, however, advances in therapy were entering clinical practice. The researchers pointed to a substantial decline in mortality from breast cancer from the early 1980s until 2013. But their conclusion was that none of the decline in mortality for breast cancer could be attributed to screening mammography when they looked at time trends; from their perspective all of the important decline in breast cancer mortality resulted from better treatment. They concluded that government programs should not support screening mammography.5
That is a recommendation that we do not support. However, we do recognize the conundrum that mammography is less sensitive among those who have dense breasts. In order to have congruent professional guidelines, we support research funding to determine which types, starting ages, and intervals of screening would be best in various patient populations. The USPSTF cites data from studies performed in the 1980s based on outdated technology; more recent (and relevant) randomized clinical trials have not been performed, and yet this information is critical to provide sufficient evidence to develop appropriate guidelines.
Our recommendations for gathering new data
The kind of data we would find most valuable would assess how different screening strategies impact overall mortality and breast cancer-specific mortality. It would require decades of follow-up—which of course means that screening technology will change over that time. A surrogate for evaluating overall survival is to look at interval cancers, which are all breast cancers diagnosed following negative mammograms and prior to the next screening. These cancers may or may not be biologically active, again focusing us on the need to look at overall survival of the patient. In addition, reducing breast cancer mortality may not reduce overall mortality, because the treatment for breast cancer may cause heart disease, or osteoporosis, or something else that impacts overall survival. These are important considerations for women and physicians who are making choices on treatment. What matters to a patient are 2 overlapping questions:
- Do I have a life-threatening condition or do I not?
- Has screening identified a condition that might lead to treatment that’s unnecessary?
The problem is that with breast cancer we can’t tell the difference. We do not understand the biological potential of a lesion when we evaluate an image on MRI, or computed tomography (CT), or mammography.
A re-look at presented data
A trial conducted by Bakker and colleagues6 was discussed by the authors of the DenseBreast-info.org quiz in which they recommended breast MRI for all women with extremely dense breasts (but no other risk factors for breast cancer) detected on screening mammograms.7 The Bakker study was large and conducted in the Netherlands. The primary outcome of the trial was to compare the incidence of interval breast cancers of women aged 50 to 75 randomly assigned to MRI versus those assigned to continued screening mammography every 2 years. Importantly, among the more than 8,000 women who were assigned to MRI, 59%, or fewer than two-thirds, chose to actually undergo MRI.
Among women randomized to MRI, 20 interval cancers were found—4 were diagnosed in those who actually had MRIs, and 16 were diagnosed among women who were randomized to MRI but didn’t undergo the study. Among women assigned to screening mammography only, 161 interval cancers were diagnosed among more than 32,000 women screened. The primary outcome findings were 2.5 interval cancers per 1,000 screenings among women randomly assigned to MRI, and 5 interval cancers per 1,000 screenings among those randomly assigned to mammography only.6
Because the trial included women aged 50 and older, we can’t apply these results to younger women, who often undergo screening mammography in the United States. In addition, the majority of the population in the Netherlands are of Western European ethnicity, a less-diverse population of women than in the United States. Furthermore, among the tumors that were detected in the MRI group, a larger proportion were DCIS, early-stage tumors, well differentiated, and hormone receptor-positive. This observation supports that many of the MRI-detected tumors were cases of overdiagnosis, or the detection of tumors destined not to cause clinical problems for the patient during her lifetime, or for which earlier diagnosis would impact survival.
We also know that treatment of these small ER-positive tumors carries risks for patients, as we may treat them by depriving a patient of estrogen for the rest of her life, with potential consequences of sexual dysfunction, osteoporosis, and perhaps cardiovascular disease depending on her age at the time of that diagnosis. Weighing the risks and benefits of not only treatment but also use of more sensitive screening techniques such as MRI is extremely important. Although Bakker and colleagues’ study results are interesting, we do not feel they support routinely recommending MRI for women found to have extremely dense breasts with mammography.
Overdiagnosis: A difficult concept
One reason overdiagnosis is so challenging to understand is that it can’t be directly measured, which makes comprehending it that much more problematic for clinicians and our patients.
One way to help grasp the overall issue is to compare screening mammography with cervical and colon cancer screening.
We are well aware that cervical cancer screening has reduced the incidence of mortality from invasive cervical cancer.8 We can argue very validly that the biggest success in any cancer screening program in history and globally has been cervical cancer screening. Our specialty, in particular, should feel proud about this. Screening colonoscopy also has repeatedly been found to reduce colon cancer mortality.9 For breast cancer, decades of media messaging have emphasized the benefits of screening mammograms; however, in contrast with cervical cancer screening and colonoscopy, screening mammography has not reduced the incidence of breast cancer presenting with metastatic or advanced disease. Danish authors pointed out in 2017 that screening mammography has not achieved the hoped for or the promised reduction in breast cancer mortality.10
A report published in the March 2022, issue of Annals of Internal Medicine used modeling techniques to estimate the incidence of overdiagnosis and concluded that, among women aged 50-74 years receiving biennial screening mammograms (consistent with USPSTF recommendations), more than 15% of screen-detected breast cancers would represent cases of overdiagnosis. Of note, the study authors found that, among screen-detected cancers, the proportion representing overdiagnosis among women in their 60s (16.7%) and early 70s (23.6%) was higher than among women in their 50s-60s (11.5%-11.6%).11
The former Chief Medical and Scientific Officer for the American Cancer Society Otis Brawley, MD, has stated that, at the same time that breast cancer screening should not be abandoned, “We must acknowledge that overdiagnosis is common. The benefits of screening have been overstated, and some patients considered as ‘cured’ from breast cancer have, in fact, been harmed by unneeded treatment.”12
“Everybody loves early detection,” said Donald Berry, PhD, from MD Anderson Cancer Center, “but it comes with harms.” He points out that mortality rates have improved for breast cancer, but he attributes it to improved treatment. “The harms [of screening] we know, but the benefits of screening are very uncertain.”13
The importance of health equity is receiving more attention. When examining equity according to breast cancer mortality, ethnic minority populations have worse cancer survival outcomes than White women; the mortality rate is 40% higher among Black women than among White women.1 Lower survival rates are also noted among lower socioeconomic groups and among women who live in rural areas. Lower survival rates among ethnic minority women are also noted for cervical and colorectal cancers.2
In the past, these disparities in mortality were attributed to the historically lower breast cancer screening rates among Black women compared with White women. However, decades of efforts to increase mammography rates have effectively addressed much of the racial/ethnic gap in screening rates.1 In fact, a 2021 study showed Black and Hispanic women to have 6% to 10% higher rates of breast, cervical, and colorectal cancer screening than White women according to US Preventive Services Task Force guidelines.2 The study authors point out that other national data have demonstrated similar results and conclude that “higher cancer mortality among racial/ethnic minority groups will not be reduced solely by increasing rates of cancer screening. Although preventive screenings and timely diagnosis are important elements of prognosis, they are just 2 elements of many along the cancer care continuum that need to be addressed to eliminate disparities in cancer mortality.”
Unfortunately, the randomized trials that have been conducted on mammography have been conducted overwhelmingly in White populations. National registry studies from the Netherlands and Sweden are not representative patient populations for the United States. Recently, the US government proposed an ambitious plan to cut cancer mortality rates and has promised vast amounts of research funding to achieve that goal.3 Hopefully, this funding will support studies which enroll diverse patient populations. We hope to gain knowledge on what elements along the cancer care continuum can be addressed to better reduce or eliminate cancer mortality inequities.
References
1. National Cancer Institute. SEER Explorer. https://seer.cancer.gov/explorer/. Accessed February 9, 2022.
2. Benavidez GA, Zgodic A, Zahnd WE, Eberth JM. Disparities in Meeting USPSTF Breast, Cervical, and Colorectal Cancer Screening Guidelines Among Women in the United States. Prev Chronic Dis. 2021;18:200315. doi: http://dx.doi.org/10.5888/pcd18.200315.
3. Stohlberg SG, Kolata G. Biden presents ambitious plan to cut cancer death rate in half. The New York Times. February 2, 2022.
Continue to: Limitations of breast MRI...
Limitations of breast MRI
Overall, MRI is a diagnostic and monitoring test. It is costlier than mammography, and because it is not recommended in guidelines as a screening modality for most women, it is not typically covered by insurance. Abbreviated (rapid) MRI is a non-standardized imaging strategy being used at a few health centers. It has a shorter protocol overall than MRI, so it takes less time than current MRI and is less expensive, but there are few data on sensitivity and specificity. It is yet to be determined which populations could benefit from this newer technology.
As mentioned, 41% of women in the Bakker et al trial who were randomly assigned to breast MRI chose not to proceed with that exam even though it would have been at no cost to them.6 Anecdotally, some patients who have undergone MRI say they would forgo it a second time as a screening modality because it was a very unpleasant, stressful experience. It’s not a perfect test, although it is more sensitive than mammography.
Other options for following up dense-breast screening. Besides MRI and abbreviated MRI, the following modalities can be used to evaluate women found to have dense breasts with screening mammograms: CT mammography with contrast, molecular breast imaging, and ultrasonography.
Screening and treatment advances
3D mammography. In the US, the great majority of screening mammography now is performed with tomosynthesis, or what our patients sometimes call 3D mammography. In fact, it is approaching standard of care. Women whose screening mammography includes tomosynthesis are less likely to experience a so-called callback for additional imaging with diagnostic mammography or breast ultrasonography.14
Liquid biopsy. A potential major advancement for making decisions about when to treat cancers in general involves determining the biological behavior of a tumor, based on analysis of either circulating tumor DNA or proteins in the blood. As more experience with this new technology accumulates, the role of liquid biopsies for breast cancer will expand.15 Liquid biopsies for screening remain investigational for now, but they hold tremendous potential.
Noninvasive proteomics. With the development of noninvasive proteomic biomarkers obtained from blood, saliva, or nipple aspiration fluid, there exists the possibility of not just evaluating an image of a tumor seen on a mammogram, but actually studying the biological characteristics of that lesion.16 The cost of this technology is far less in terms of resources than MRI or molecular-based imaging, and actually reveals the flaws with using image-based screening. With proteomics, we can tell whether or not a lump is generating proteins that are going to make that disease biologically meaningful, and treatment decisions can be based on that information. This idea has the potential to disrupt our current breast cancer screening paradigm.
Advocacy’s role in mandating legislation
Many advocacy groups lobby on Capitol Hill for legislation related to health care, but we don’t feel that is the best way to make scientific decisions, and it’s not the way to do medicine. Passionate people, who truly believe that their outcome would have been different had something else been done, have every right to advocate, and should. However, without longer-term data focusing on breast cancer and overall mortality, rather than surrogate outcomes like interval cancers, it is not clear that routinely recommending supplemental MRI will improve survival for women with extremely dense breasts. Unfortunately, overall, earlier diagnosis of highly aggressive breast cancer tumors does not result in better outcomes for patients. ●
- American College of Obstetricians and Gynecologists. Practice Bulletin number 179: breast cancer risk assessment and screening in average-risk women. Obstet Gynecol. 2017;130: e1-e16. doi: 10.1097/AOG.0000000000002158.
- Sui AL, U.S. Preventive Services Task Force. Screening for breast cancer: U.S. Preventive Services Task Force recommendation statement. Ann Intern Med. 2016;164:279-296. doi: 10.7326/M15-2886.
- The American Society of Breast Surgeons. Position statement on screening mammography. https://www.breastsurgeons.org/docs /statements/Position-Statement-on-ScreeningMammography.pdf. Accessed February 15, 2022.
- Monticciolo DL, Malak SF, Friedewald SM, et al. Breast cancer screening recommendations inclusive of all women at average-risk: update from the ACR and Society of Breast Imaging. J Am College Radiol. 2021;18:1280-1288.
- Burton R, Stevenson C. Assessment of breast cancer mortality trends associated with mammographic screening and adjuvant therapy from 1986 to 2013 in the state of Victoria, Australia. JAMA Netw Open. 2020;3:e208249.
- Bakker MF, de Lange SV, Pijnappel RM, et al. Supplemental MRI screening for women with extremely dense breast tissue. N Engl J Med. 2019;381:2091-2102. doi: 10.1056/NEJMoa1903986.
- Seitzman R, Berg W. Average-risk women with dense breasts—what breast screening is appropriate? OBG Manag. 2021;33:18-19. doi: 10.12788/obgm.0155.
- Gopalani SV, Janitz AE, Campbell JE. Cervical cancer incidence and mortality among non-hispanic African American and White women, United States, 1999-2015. J Natl Med Assoc. 2020;112:632-638. doi: 10.1016 /j.jnma.2020.06.007.
- Niikura R, Hirata Y, Suzuki N, et al. Colonoscopy reduces colorectal cancer mortality: a multicenter, long-term, colonoscopy-based cohort study. PLoS One. 2017;12:e0185294.
- Jørgensen KJ, Gøtzsche PC, Kalager M, et al. Breast cancer screening in Denmark. Ann Intern Med. 2017;167:524. doi: 10.7326/L17-0270.
- Ryser MD, Lange J, Inoue IL, et al. Estimation of breast cancer overdiagnosis in a U.S. breast screening cohort. Ann Intern Med. 2022 March 1. doi: 10.7326/M21-3577.
- Brawley OW. Accepting the existence of breast cancer overdiagnosis. Ann Intern Med. 2017;166:364-365. doi:10.7326/M16-2850.
- Stohlberg SG, Kolata G. Biden presents ambitious plan to cut cancer death rate in half. The New York Times. February 2, 2022.
- Conant EF, Barlow WE, Herschorn SD, et al. Association of digital breast tomosynthesis vs digital mammography with cancer detection and recall rates by age and breast density. JAMA Oncol. 2019;5:635-642. doi: 10.1001 /jamaoncol.2018.7078.
- Tay TK, Tan PH. Liquid biopsy in breast cancer: a focused review. Arch Pathol Lab Med. 2021;145: 678-686. doi: 10.5858/arpa.2019-0559-RA.
- Debald M, Wolgarten M, Walgenbach-Brunagel G, et al. Non-invasive proteomics—thinking about personalized breast cancer screening and treatment. EPMA J. 2010;1:413-420. doi: 10.1007 /s13167-010-0039-9.
We have been interested in the quiz series focused on breast cancer screening for women with dense breasts presented in
The concerns with breast cancer in particular
Breast cancer is not cervical cancer. It isn’t one disease. It is a multitude of diseases that happen to show up in the breast. Some are relatively slow-growing—the kinds of cancers that lend themselves to screening and to early intervention. But other cancers are rapidly-growing; they show up no matter how often or what modality we use for screening. Our goal should be to find an approach to screening that can diagnose breast cancer at a stage where we can intervene and positively impact breast cancer specific and overall mortality.
Screening guidelines vary
The variety of screening guidelines published by different professional organizations reflect differing assumptions and sets of values related to the early diagnosis and treatment of breast cancer. (For a comprehensive table of current screening guidelines, see https://www.cdc.gov/cancer/breast/pdf/breast-cancer-screening-guidelines-508.pdf.)
ACOG’s approach—to offer screening at age 40 but to begin by at least age 50 and, through shared decision making with the patient, screen every 1 or 2 years—is focused on capturing as many cases as we can identify, while minimizing the harms of false-positives.1 The perspective of the US Preventive Services Task Force (USPSTF) recommendations (to screen every 2 years beginning at age 50) is at the population level, a cost-effective approach that will have the greatest benefit while minimizing harms in the population at large.2 The American Society of Breast Surgeons recommends screening to begin by age 40.3 Like the breast surgeons, radiologists dedicated to breast imaging are focused on an individual rather than a population level. They strive to identify each and every instance of possible cancer, and therefore recommend annual screening beginning at age 40.4 However, with more aggressive screening in average-risk women many cases of ductal carcinoma in situ (DCIS) are identified—a lesion that, if not detected, may not impact the woman’s health during her lifetime—representing what some might call “overdiagnosis.” Yet there may be some instances in which the DCIS might affect an individual woman’s health. Unfortunately, we can’t prospectively distinguish between the first and the second types of cases.
We follow American College of Obstetricians and Gynecologists and US Preventive Services Task Force guidelines in discussing screening (both its hazards and benefits) with our average-risk patients beginning at age 40. We talk about risk factors for breast cancer, including breast density, but let patients know that no specific additional imaging is advised, and that density is more common in younger women (one consideration in earlier screening) and is quite common in general. Although we do not send follow-up letters to patients with dense breasts, we do educate our staff so that they can respond appropriately should patients call with questions.
Of course, we all bring to the table values that will impact the decisions that we make for ourselves and for our patients. What an ObGyn might suggest may differ from what a radiologist might suggest. Although we follow recommendations made by the radiologist at screening, an ObGyn wants to take care of the whole human being. We are concerned with bones, heart, everything about the patient, so we approach a patient in a different way. These priorities are reflected in the current varying breast cancer screening guidelines.
Continue to: Research on breast cancer screening varies by design...
Research on breast cancer screening varies by design
There has not been a randomized clinical trial conducted on screening mammography since the days of the analog mammogram. The research that has been conducted is difficult to compare due to variations in screening ages and intervals, technology sensitivity, and patient adherence with recommended screening. Treatments for breast cancer also have changed dramatically over time, so the findings of older studies may no longer be relevant to current breast cancer screening. The kind of analysis that needs to be done is an interrupted time series, where you can look at the trajectory of breast cancer survival and whether screening mammography shifts that survival in any way.
One specific study from Australia measured the impact of newer available breast cancer treatments, including tamoxifen for women with receptor-positive tumors and newer chemotherapy strategies.5 The authors analyzed screening mammography trends in one large province where women aged 50 to 69 were offered biennial screening. Trends from the 1990s showed that more women were being screened over time. Simultaneously, however, advances in therapy were entering clinical practice. The researchers pointed to a substantial decline in mortality from breast cancer from the early 1980s until 2013. But their conclusion was that none of the decline in mortality for breast cancer could be attributed to screening mammography when they looked at time trends; from their perspective all of the important decline in breast cancer mortality resulted from better treatment. They concluded that government programs should not support screening mammography.5
That is a recommendation that we do not support. However, we do recognize the conundrum that mammography is less sensitive among those who have dense breasts. In order to have congruent professional guidelines, we support research funding to determine which types, starting ages, and intervals of screening would be best in various patient populations. The USPSTF cites data from studies performed in the 1980s based on outdated technology; more recent (and relevant) randomized clinical trials have not been performed, and yet this information is critical to provide sufficient evidence to develop appropriate guidelines.
Our recommendations for gathering new data
The kind of data we would find most valuable would assess how different screening strategies impact overall mortality and breast cancer-specific mortality. It would require decades of follow-up—which of course means that screening technology will change over that time. A surrogate for evaluating overall survival is to look at interval cancers, which are all breast cancers diagnosed following negative mammograms and prior to the next screening. These cancers may or may not be biologically active, again focusing us on the need to look at overall survival of the patient. In addition, reducing breast cancer mortality may not reduce overall mortality, because the treatment for breast cancer may cause heart disease, or osteoporosis, or something else that impacts overall survival. These are important considerations for women and physicians who are making choices on treatment. What matters to a patient are 2 overlapping questions:
- Do I have a life-threatening condition or do I not?
- Has screening identified a condition that might lead to treatment that’s unnecessary?
The problem is that with breast cancer we can’t tell the difference. We do not understand the biological potential of a lesion when we evaluate an image on MRI, or computed tomography (CT), or mammography.
A re-look at presented data
A trial conducted by Bakker and colleagues6 was discussed by the authors of the DenseBreast-info.org quiz in which they recommended breast MRI for all women with extremely dense breasts (but no other risk factors for breast cancer) detected on screening mammograms.7 The Bakker study was large and conducted in the Netherlands. The primary outcome of the trial was to compare the incidence of interval breast cancers of women aged 50 to 75 randomly assigned to MRI versus those assigned to continued screening mammography every 2 years. Importantly, among the more than 8,000 women who were assigned to MRI, 59%, or fewer than two-thirds, chose to actually undergo MRI.
Among women randomized to MRI, 20 interval cancers were found—4 were diagnosed in those who actually had MRIs, and 16 were diagnosed among women who were randomized to MRI but didn’t undergo the study. Among women assigned to screening mammography only, 161 interval cancers were diagnosed among more than 32,000 women screened. The primary outcome findings were 2.5 interval cancers per 1,000 screenings among women randomly assigned to MRI, and 5 interval cancers per 1,000 screenings among those randomly assigned to mammography only.6
Because the trial included women aged 50 and older, we can’t apply these results to younger women, who often undergo screening mammography in the United States. In addition, the majority of the population in the Netherlands are of Western European ethnicity, a less-diverse population of women than in the United States. Furthermore, among the tumors that were detected in the MRI group, a larger proportion were DCIS, early-stage tumors, well differentiated, and hormone receptor-positive. This observation supports that many of the MRI-detected tumors were cases of overdiagnosis, or the detection of tumors destined not to cause clinical problems for the patient during her lifetime, or for which earlier diagnosis would impact survival.
We also know that treatment of these small ER-positive tumors carries risks for patients, as we may treat them by depriving a patient of estrogen for the rest of her life, with potential consequences of sexual dysfunction, osteoporosis, and perhaps cardiovascular disease depending on her age at the time of that diagnosis. Weighing the risks and benefits of not only treatment but also use of more sensitive screening techniques such as MRI is extremely important. Although Bakker and colleagues’ study results are interesting, we do not feel they support routinely recommending MRI for women found to have extremely dense breasts with mammography.
Overdiagnosis: A difficult concept
One reason overdiagnosis is so challenging to understand is that it can’t be directly measured, which makes comprehending it that much more problematic for clinicians and our patients.
One way to help grasp the overall issue is to compare screening mammography with cervical and colon cancer screening.
We are well aware that cervical cancer screening has reduced the incidence of mortality from invasive cervical cancer.8 We can argue very validly that the biggest success in any cancer screening program in history and globally has been cervical cancer screening. Our specialty, in particular, should feel proud about this. Screening colonoscopy also has repeatedly been found to reduce colon cancer mortality.9 For breast cancer, decades of media messaging have emphasized the benefits of screening mammograms; however, in contrast with cervical cancer screening and colonoscopy, screening mammography has not reduced the incidence of breast cancer presenting with metastatic or advanced disease. Danish authors pointed out in 2017 that screening mammography has not achieved the hoped for or the promised reduction in breast cancer mortality.10
A report published in the March 2022, issue of Annals of Internal Medicine used modeling techniques to estimate the incidence of overdiagnosis and concluded that, among women aged 50-74 years receiving biennial screening mammograms (consistent with USPSTF recommendations), more than 15% of screen-detected breast cancers would represent cases of overdiagnosis. Of note, the study authors found that, among screen-detected cancers, the proportion representing overdiagnosis among women in their 60s (16.7%) and early 70s (23.6%) was higher than among women in their 50s-60s (11.5%-11.6%).11
The former Chief Medical and Scientific Officer for the American Cancer Society Otis Brawley, MD, has stated that, at the same time that breast cancer screening should not be abandoned, “We must acknowledge that overdiagnosis is common. The benefits of screening have been overstated, and some patients considered as ‘cured’ from breast cancer have, in fact, been harmed by unneeded treatment.”12
“Everybody loves early detection,” said Donald Berry, PhD, from MD Anderson Cancer Center, “but it comes with harms.” He points out that mortality rates have improved for breast cancer, but he attributes it to improved treatment. “The harms [of screening] we know, but the benefits of screening are very uncertain.”13
The importance of health equity is receiving more attention. When examining equity according to breast cancer mortality, ethnic minority populations have worse cancer survival outcomes than White women; the mortality rate is 40% higher among Black women than among White women.1 Lower survival rates are also noted among lower socioeconomic groups and among women who live in rural areas. Lower survival rates among ethnic minority women are also noted for cervical and colorectal cancers.2
In the past, these disparities in mortality were attributed to the historically lower breast cancer screening rates among Black women compared with White women. However, decades of efforts to increase mammography rates have effectively addressed much of the racial/ethnic gap in screening rates.1 In fact, a 2021 study showed Black and Hispanic women to have 6% to 10% higher rates of breast, cervical, and colorectal cancer screening than White women according to US Preventive Services Task Force guidelines.2 The study authors point out that other national data have demonstrated similar results and conclude that “higher cancer mortality among racial/ethnic minority groups will not be reduced solely by increasing rates of cancer screening. Although preventive screenings and timely diagnosis are important elements of prognosis, they are just 2 elements of many along the cancer care continuum that need to be addressed to eliminate disparities in cancer mortality.”
Unfortunately, the randomized trials that have been conducted on mammography have been conducted overwhelmingly in White populations. National registry studies from the Netherlands and Sweden are not representative patient populations for the United States. Recently, the US government proposed an ambitious plan to cut cancer mortality rates and has promised vast amounts of research funding to achieve that goal.3 Hopefully, this funding will support studies which enroll diverse patient populations. We hope to gain knowledge on what elements along the cancer care continuum can be addressed to better reduce or eliminate cancer mortality inequities.
References
1. National Cancer Institute. SEER Explorer. https://seer.cancer.gov/explorer/. Accessed February 9, 2022.
2. Benavidez GA, Zgodic A, Zahnd WE, Eberth JM. Disparities in Meeting USPSTF Breast, Cervical, and Colorectal Cancer Screening Guidelines Among Women in the United States. Prev Chronic Dis. 2021;18:200315. doi: http://dx.doi.org/10.5888/pcd18.200315.
3. Stohlberg SG, Kolata G. Biden presents ambitious plan to cut cancer death rate in half. The New York Times. February 2, 2022.
Continue to: Limitations of breast MRI...
Limitations of breast MRI
Overall, MRI is a diagnostic and monitoring test. It is costlier than mammography, and because it is not recommended in guidelines as a screening modality for most women, it is not typically covered by insurance. Abbreviated (rapid) MRI is a non-standardized imaging strategy being used at a few health centers. It has a shorter protocol overall than MRI, so it takes less time than current MRI and is less expensive, but there are few data on sensitivity and specificity. It is yet to be determined which populations could benefit from this newer technology.
As mentioned, 41% of women in the Bakker et al trial who were randomly assigned to breast MRI chose not to proceed with that exam even though it would have been at no cost to them.6 Anecdotally, some patients who have undergone MRI say they would forgo it a second time as a screening modality because it was a very unpleasant, stressful experience. It’s not a perfect test, although it is more sensitive than mammography.
Other options for following up dense-breast screening. Besides MRI and abbreviated MRI, the following modalities can be used to evaluate women found to have dense breasts with screening mammograms: CT mammography with contrast, molecular breast imaging, and ultrasonography.
Screening and treatment advances
3D mammography. In the US, the great majority of screening mammography now is performed with tomosynthesis, or what our patients sometimes call 3D mammography. In fact, it is approaching standard of care. Women whose screening mammography includes tomosynthesis are less likely to experience a so-called callback for additional imaging with diagnostic mammography or breast ultrasonography.14
Liquid biopsy. A potential major advancement for making decisions about when to treat cancers in general involves determining the biological behavior of a tumor, based on analysis of either circulating tumor DNA or proteins in the blood. As more experience with this new technology accumulates, the role of liquid biopsies for breast cancer will expand.15 Liquid biopsies for screening remain investigational for now, but they hold tremendous potential.
Noninvasive proteomics. With the development of noninvasive proteomic biomarkers obtained from blood, saliva, or nipple aspiration fluid, there exists the possibility of not just evaluating an image of a tumor seen on a mammogram, but actually studying the biological characteristics of that lesion.16 The cost of this technology is far less in terms of resources than MRI or molecular-based imaging, and actually reveals the flaws with using image-based screening. With proteomics, we can tell whether or not a lump is generating proteins that are going to make that disease biologically meaningful, and treatment decisions can be based on that information. This idea has the potential to disrupt our current breast cancer screening paradigm.
Advocacy’s role in mandating legislation
Many advocacy groups lobby on Capitol Hill for legislation related to health care, but we don’t feel that is the best way to make scientific decisions, and it’s not the way to do medicine. Passionate people, who truly believe that their outcome would have been different had something else been done, have every right to advocate, and should. However, without longer-term data focusing on breast cancer and overall mortality, rather than surrogate outcomes like interval cancers, it is not clear that routinely recommending supplemental MRI will improve survival for women with extremely dense breasts. Unfortunately, overall, earlier diagnosis of highly aggressive breast cancer tumors does not result in better outcomes for patients. ●
We have been interested in the quiz series focused on breast cancer screening for women with dense breasts presented in
The concerns with breast cancer in particular
Breast cancer is not cervical cancer. It isn’t one disease. It is a multitude of diseases that happen to show up in the breast. Some are relatively slow-growing—the kinds of cancers that lend themselves to screening and to early intervention. But other cancers are rapidly-growing; they show up no matter how often or what modality we use for screening. Our goal should be to find an approach to screening that can diagnose breast cancer at a stage where we can intervene and positively impact breast cancer specific and overall mortality.
Screening guidelines vary
The variety of screening guidelines published by different professional organizations reflect differing assumptions and sets of values related to the early diagnosis and treatment of breast cancer. (For a comprehensive table of current screening guidelines, see https://www.cdc.gov/cancer/breast/pdf/breast-cancer-screening-guidelines-508.pdf.)
ACOG’s approach—to offer screening at age 40 but to begin by at least age 50 and, through shared decision making with the patient, screen every 1 or 2 years—is focused on capturing as many cases as we can identify, while minimizing the harms of false-positives.1 The perspective of the US Preventive Services Task Force (USPSTF) recommendations (to screen every 2 years beginning at age 50) is at the population level, a cost-effective approach that will have the greatest benefit while minimizing harms in the population at large.2 The American Society of Breast Surgeons recommends screening to begin by age 40.3 Like the breast surgeons, radiologists dedicated to breast imaging are focused on an individual rather than a population level. They strive to identify each and every instance of possible cancer, and therefore recommend annual screening beginning at age 40.4 However, with more aggressive screening in average-risk women many cases of ductal carcinoma in situ (DCIS) are identified—a lesion that, if not detected, may not impact the woman’s health during her lifetime—representing what some might call “overdiagnosis.” Yet there may be some instances in which the DCIS might affect an individual woman’s health. Unfortunately, we can’t prospectively distinguish between the first and the second types of cases.
We follow American College of Obstetricians and Gynecologists and US Preventive Services Task Force guidelines in discussing screening (both its hazards and benefits) with our average-risk patients beginning at age 40. We talk about risk factors for breast cancer, including breast density, but let patients know that no specific additional imaging is advised, and that density is more common in younger women (one consideration in earlier screening) and is quite common in general. Although we do not send follow-up letters to patients with dense breasts, we do educate our staff so that they can respond appropriately should patients call with questions.
Of course, we all bring to the table values that will impact the decisions that we make for ourselves and for our patients. What an ObGyn might suggest may differ from what a radiologist might suggest. Although we follow recommendations made by the radiologist at screening, an ObGyn wants to take care of the whole human being. We are concerned with bones, heart, everything about the patient, so we approach a patient in a different way. These priorities are reflected in the current varying breast cancer screening guidelines.
Continue to: Research on breast cancer screening varies by design...
Research on breast cancer screening varies by design
There has not been a randomized clinical trial conducted on screening mammography since the days of the analog mammogram. The research that has been conducted is difficult to compare due to variations in screening ages and intervals, technology sensitivity, and patient adherence with recommended screening. Treatments for breast cancer also have changed dramatically over time, so the findings of older studies may no longer be relevant to current breast cancer screening. The kind of analysis that needs to be done is an interrupted time series, where you can look at the trajectory of breast cancer survival and whether screening mammography shifts that survival in any way.
One specific study from Australia measured the impact of newer available breast cancer treatments, including tamoxifen for women with receptor-positive tumors and newer chemotherapy strategies.5 The authors analyzed screening mammography trends in one large province where women aged 50 to 69 were offered biennial screening. Trends from the 1990s showed that more women were being screened over time. Simultaneously, however, advances in therapy were entering clinical practice. The researchers pointed to a substantial decline in mortality from breast cancer from the early 1980s until 2013. But their conclusion was that none of the decline in mortality for breast cancer could be attributed to screening mammography when they looked at time trends; from their perspective all of the important decline in breast cancer mortality resulted from better treatment. They concluded that government programs should not support screening mammography.5
That is a recommendation that we do not support. However, we do recognize the conundrum that mammography is less sensitive among those who have dense breasts. In order to have congruent professional guidelines, we support research funding to determine which types, starting ages, and intervals of screening would be best in various patient populations. The USPSTF cites data from studies performed in the 1980s based on outdated technology; more recent (and relevant) randomized clinical trials have not been performed, and yet this information is critical to provide sufficient evidence to develop appropriate guidelines.
Our recommendations for gathering new data
The kind of data we would find most valuable would assess how different screening strategies impact overall mortality and breast cancer-specific mortality. It would require decades of follow-up—which of course means that screening technology will change over that time. A surrogate for evaluating overall survival is to look at interval cancers, which are all breast cancers diagnosed following negative mammograms and prior to the next screening. These cancers may or may not be biologically active, again focusing us on the need to look at overall survival of the patient. In addition, reducing breast cancer mortality may not reduce overall mortality, because the treatment for breast cancer may cause heart disease, or osteoporosis, or something else that impacts overall survival. These are important considerations for women and physicians who are making choices on treatment. What matters to a patient are 2 overlapping questions:
- Do I have a life-threatening condition or do I not?
- Has screening identified a condition that might lead to treatment that’s unnecessary?
The problem is that with breast cancer we can’t tell the difference. We do not understand the biological potential of a lesion when we evaluate an image on MRI, or computed tomography (CT), or mammography.
A re-look at presented data
A trial conducted by Bakker and colleagues6 was discussed by the authors of the DenseBreast-info.org quiz in which they recommended breast MRI for all women with extremely dense breasts (but no other risk factors for breast cancer) detected on screening mammograms.7 The Bakker study was large and conducted in the Netherlands. The primary outcome of the trial was to compare the incidence of interval breast cancers of women aged 50 to 75 randomly assigned to MRI versus those assigned to continued screening mammography every 2 years. Importantly, among the more than 8,000 women who were assigned to MRI, 59%, or fewer than two-thirds, chose to actually undergo MRI.
Among women randomized to MRI, 20 interval cancers were found—4 were diagnosed in those who actually had MRIs, and 16 were diagnosed among women who were randomized to MRI but didn’t undergo the study. Among women assigned to screening mammography only, 161 interval cancers were diagnosed among more than 32,000 women screened. The primary outcome findings were 2.5 interval cancers per 1,000 screenings among women randomly assigned to MRI, and 5 interval cancers per 1,000 screenings among those randomly assigned to mammography only.6
Because the trial included women aged 50 and older, we can’t apply these results to younger women, who often undergo screening mammography in the United States. In addition, the majority of the population in the Netherlands are of Western European ethnicity, a less-diverse population of women than in the United States. Furthermore, among the tumors that were detected in the MRI group, a larger proportion were DCIS, early-stage tumors, well differentiated, and hormone receptor-positive. This observation supports that many of the MRI-detected tumors were cases of overdiagnosis, or the detection of tumors destined not to cause clinical problems for the patient during her lifetime, or for which earlier diagnosis would impact survival.
We also know that treatment of these small ER-positive tumors carries risks for patients, as we may treat them by depriving a patient of estrogen for the rest of her life, with potential consequences of sexual dysfunction, osteoporosis, and perhaps cardiovascular disease depending on her age at the time of that diagnosis. Weighing the risks and benefits of not only treatment but also use of more sensitive screening techniques such as MRI is extremely important. Although Bakker and colleagues’ study results are interesting, we do not feel they support routinely recommending MRI for women found to have extremely dense breasts with mammography.
Overdiagnosis: A difficult concept
One reason overdiagnosis is so challenging to understand is that it can’t be directly measured, which makes comprehending it that much more problematic for clinicians and our patients.
One way to help grasp the overall issue is to compare screening mammography with cervical and colon cancer screening.
We are well aware that cervical cancer screening has reduced the incidence of mortality from invasive cervical cancer.8 We can argue very validly that the biggest success in any cancer screening program in history and globally has been cervical cancer screening. Our specialty, in particular, should feel proud about this. Screening colonoscopy also has repeatedly been found to reduce colon cancer mortality.9 For breast cancer, decades of media messaging have emphasized the benefits of screening mammograms; however, in contrast with cervical cancer screening and colonoscopy, screening mammography has not reduced the incidence of breast cancer presenting with metastatic or advanced disease. Danish authors pointed out in 2017 that screening mammography has not achieved the hoped for or the promised reduction in breast cancer mortality.10
A report published in the March 2022, issue of Annals of Internal Medicine used modeling techniques to estimate the incidence of overdiagnosis and concluded that, among women aged 50-74 years receiving biennial screening mammograms (consistent with USPSTF recommendations), more than 15% of screen-detected breast cancers would represent cases of overdiagnosis. Of note, the study authors found that, among screen-detected cancers, the proportion representing overdiagnosis among women in their 60s (16.7%) and early 70s (23.6%) was higher than among women in their 50s-60s (11.5%-11.6%).11
The former Chief Medical and Scientific Officer for the American Cancer Society Otis Brawley, MD, has stated that, at the same time that breast cancer screening should not be abandoned, “We must acknowledge that overdiagnosis is common. The benefits of screening have been overstated, and some patients considered as ‘cured’ from breast cancer have, in fact, been harmed by unneeded treatment.”12
“Everybody loves early detection,” said Donald Berry, PhD, from MD Anderson Cancer Center, “but it comes with harms.” He points out that mortality rates have improved for breast cancer, but he attributes it to improved treatment. “The harms [of screening] we know, but the benefits of screening are very uncertain.”13
The importance of health equity is receiving more attention. When examining equity according to breast cancer mortality, ethnic minority populations have worse cancer survival outcomes than White women; the mortality rate is 40% higher among Black women than among White women.1 Lower survival rates are also noted among lower socioeconomic groups and among women who live in rural areas. Lower survival rates among ethnic minority women are also noted for cervical and colorectal cancers.2
In the past, these disparities in mortality were attributed to the historically lower breast cancer screening rates among Black women compared with White women. However, decades of efforts to increase mammography rates have effectively addressed much of the racial/ethnic gap in screening rates.1 In fact, a 2021 study showed Black and Hispanic women to have 6% to 10% higher rates of breast, cervical, and colorectal cancer screening than White women according to US Preventive Services Task Force guidelines.2 The study authors point out that other national data have demonstrated similar results and conclude that “higher cancer mortality among racial/ethnic minority groups will not be reduced solely by increasing rates of cancer screening. Although preventive screenings and timely diagnosis are important elements of prognosis, they are just 2 elements of many along the cancer care continuum that need to be addressed to eliminate disparities in cancer mortality.”
Unfortunately, the randomized trials that have been conducted on mammography have been conducted overwhelmingly in White populations. National registry studies from the Netherlands and Sweden are not representative patient populations for the United States. Recently, the US government proposed an ambitious plan to cut cancer mortality rates and has promised vast amounts of research funding to achieve that goal.3 Hopefully, this funding will support studies which enroll diverse patient populations. We hope to gain knowledge on what elements along the cancer care continuum can be addressed to better reduce or eliminate cancer mortality inequities.
References
1. National Cancer Institute. SEER Explorer. https://seer.cancer.gov/explorer/. Accessed February 9, 2022.
2. Benavidez GA, Zgodic A, Zahnd WE, Eberth JM. Disparities in Meeting USPSTF Breast, Cervical, and Colorectal Cancer Screening Guidelines Among Women in the United States. Prev Chronic Dis. 2021;18:200315. doi: http://dx.doi.org/10.5888/pcd18.200315.
3. Stohlberg SG, Kolata G. Biden presents ambitious plan to cut cancer death rate in half. The New York Times. February 2, 2022.
Continue to: Limitations of breast MRI...
Limitations of breast MRI
Overall, MRI is a diagnostic and monitoring test. It is costlier than mammography, and because it is not recommended in guidelines as a screening modality for most women, it is not typically covered by insurance. Abbreviated (rapid) MRI is a non-standardized imaging strategy being used at a few health centers. It has a shorter protocol overall than MRI, so it takes less time than current MRI and is less expensive, but there are few data on sensitivity and specificity. It is yet to be determined which populations could benefit from this newer technology.
As mentioned, 41% of women in the Bakker et al trial who were randomly assigned to breast MRI chose not to proceed with that exam even though it would have been at no cost to them.6 Anecdotally, some patients who have undergone MRI say they would forgo it a second time as a screening modality because it was a very unpleasant, stressful experience. It’s not a perfect test, although it is more sensitive than mammography.
Other options for following up dense-breast screening. Besides MRI and abbreviated MRI, the following modalities can be used to evaluate women found to have dense breasts with screening mammograms: CT mammography with contrast, molecular breast imaging, and ultrasonography.
Screening and treatment advances
3D mammography. In the US, the great majority of screening mammography now is performed with tomosynthesis, or what our patients sometimes call 3D mammography. In fact, it is approaching standard of care. Women whose screening mammography includes tomosynthesis are less likely to experience a so-called callback for additional imaging with diagnostic mammography or breast ultrasonography.14
Liquid biopsy. A potential major advancement for making decisions about when to treat cancers in general involves determining the biological behavior of a tumor, based on analysis of either circulating tumor DNA or proteins in the blood. As more experience with this new technology accumulates, the role of liquid biopsies for breast cancer will expand.15 Liquid biopsies for screening remain investigational for now, but they hold tremendous potential.
Noninvasive proteomics. With the development of noninvasive proteomic biomarkers obtained from blood, saliva, or nipple aspiration fluid, there exists the possibility of not just evaluating an image of a tumor seen on a mammogram, but actually studying the biological characteristics of that lesion.16 The cost of this technology is far less in terms of resources than MRI or molecular-based imaging, and actually reveals the flaws with using image-based screening. With proteomics, we can tell whether or not a lump is generating proteins that are going to make that disease biologically meaningful, and treatment decisions can be based on that information. This idea has the potential to disrupt our current breast cancer screening paradigm.
Advocacy’s role in mandating legislation
Many advocacy groups lobby on Capitol Hill for legislation related to health care, but we don’t feel that is the best way to make scientific decisions, and it’s not the way to do medicine. Passionate people, who truly believe that their outcome would have been different had something else been done, have every right to advocate, and should. However, without longer-term data focusing on breast cancer and overall mortality, rather than surrogate outcomes like interval cancers, it is not clear that routinely recommending supplemental MRI will improve survival for women with extremely dense breasts. Unfortunately, overall, earlier diagnosis of highly aggressive breast cancer tumors does not result in better outcomes for patients. ●
- American College of Obstetricians and Gynecologists. Practice Bulletin number 179: breast cancer risk assessment and screening in average-risk women. Obstet Gynecol. 2017;130: e1-e16. doi: 10.1097/AOG.0000000000002158.
- Sui AL, U.S. Preventive Services Task Force. Screening for breast cancer: U.S. Preventive Services Task Force recommendation statement. Ann Intern Med. 2016;164:279-296. doi: 10.7326/M15-2886.
- The American Society of Breast Surgeons. Position statement on screening mammography. https://www.breastsurgeons.org/docs /statements/Position-Statement-on-ScreeningMammography.pdf. Accessed February 15, 2022.
- Monticciolo DL, Malak SF, Friedewald SM, et al. Breast cancer screening recommendations inclusive of all women at average-risk: update from the ACR and Society of Breast Imaging. J Am College Radiol. 2021;18:1280-1288.
- Burton R, Stevenson C. Assessment of breast cancer mortality trends associated with mammographic screening and adjuvant therapy from 1986 to 2013 in the state of Victoria, Australia. JAMA Netw Open. 2020;3:e208249.
- Bakker MF, de Lange SV, Pijnappel RM, et al. Supplemental MRI screening for women with extremely dense breast tissue. N Engl J Med. 2019;381:2091-2102. doi: 10.1056/NEJMoa1903986.
- Seitzman R, Berg W. Average-risk women with dense breasts—what breast screening is appropriate? OBG Manag. 2021;33:18-19. doi: 10.12788/obgm.0155.
- Gopalani SV, Janitz AE, Campbell JE. Cervical cancer incidence and mortality among non-hispanic African American and White women, United States, 1999-2015. J Natl Med Assoc. 2020;112:632-638. doi: 10.1016 /j.jnma.2020.06.007.
- Niikura R, Hirata Y, Suzuki N, et al. Colonoscopy reduces colorectal cancer mortality: a multicenter, long-term, colonoscopy-based cohort study. PLoS One. 2017;12:e0185294.
- Jørgensen KJ, Gøtzsche PC, Kalager M, et al. Breast cancer screening in Denmark. Ann Intern Med. 2017;167:524. doi: 10.7326/L17-0270.
- Ryser MD, Lange J, Inoue IL, et al. Estimation of breast cancer overdiagnosis in a U.S. breast screening cohort. Ann Intern Med. 2022 March 1. doi: 10.7326/M21-3577.
- Brawley OW. Accepting the existence of breast cancer overdiagnosis. Ann Intern Med. 2017;166:364-365. doi:10.7326/M16-2850.
- Stohlberg SG, Kolata G. Biden presents ambitious plan to cut cancer death rate in half. The New York Times. February 2, 2022.
- Conant EF, Barlow WE, Herschorn SD, et al. Association of digital breast tomosynthesis vs digital mammography with cancer detection and recall rates by age and breast density. JAMA Oncol. 2019;5:635-642. doi: 10.1001 /jamaoncol.2018.7078.
- Tay TK, Tan PH. Liquid biopsy in breast cancer: a focused review. Arch Pathol Lab Med. 2021;145: 678-686. doi: 10.5858/arpa.2019-0559-RA.
- Debald M, Wolgarten M, Walgenbach-Brunagel G, et al. Non-invasive proteomics—thinking about personalized breast cancer screening and treatment. EPMA J. 2010;1:413-420. doi: 10.1007 /s13167-010-0039-9.
- American College of Obstetricians and Gynecologists. Practice Bulletin number 179: breast cancer risk assessment and screening in average-risk women. Obstet Gynecol. 2017;130: e1-e16. doi: 10.1097/AOG.0000000000002158.
- Sui AL, U.S. Preventive Services Task Force. Screening for breast cancer: U.S. Preventive Services Task Force recommendation statement. Ann Intern Med. 2016;164:279-296. doi: 10.7326/M15-2886.
- The American Society of Breast Surgeons. Position statement on screening mammography. https://www.breastsurgeons.org/docs /statements/Position-Statement-on-ScreeningMammography.pdf. Accessed February 15, 2022.
- Monticciolo DL, Malak SF, Friedewald SM, et al. Breast cancer screening recommendations inclusive of all women at average-risk: update from the ACR and Society of Breast Imaging. J Am College Radiol. 2021;18:1280-1288.
- Burton R, Stevenson C. Assessment of breast cancer mortality trends associated with mammographic screening and adjuvant therapy from 1986 to 2013 in the state of Victoria, Australia. JAMA Netw Open. 2020;3:e208249.
- Bakker MF, de Lange SV, Pijnappel RM, et al. Supplemental MRI screening for women with extremely dense breast tissue. N Engl J Med. 2019;381:2091-2102. doi: 10.1056/NEJMoa1903986.
- Seitzman R, Berg W. Average-risk women with dense breasts—what breast screening is appropriate? OBG Manag. 2021;33:18-19. doi: 10.12788/obgm.0155.
- Gopalani SV, Janitz AE, Campbell JE. Cervical cancer incidence and mortality among non-hispanic African American and White women, United States, 1999-2015. J Natl Med Assoc. 2020;112:632-638. doi: 10.1016 /j.jnma.2020.06.007.
- Niikura R, Hirata Y, Suzuki N, et al. Colonoscopy reduces colorectal cancer mortality: a multicenter, long-term, colonoscopy-based cohort study. PLoS One. 2017;12:e0185294.
- Jørgensen KJ, Gøtzsche PC, Kalager M, et al. Breast cancer screening in Denmark. Ann Intern Med. 2017;167:524. doi: 10.7326/L17-0270.
- Ryser MD, Lange J, Inoue IL, et al. Estimation of breast cancer overdiagnosis in a U.S. breast screening cohort. Ann Intern Med. 2022 March 1. doi: 10.7326/M21-3577.
- Brawley OW. Accepting the existence of breast cancer overdiagnosis. Ann Intern Med. 2017;166:364-365. doi:10.7326/M16-2850.
- Stohlberg SG, Kolata G. Biden presents ambitious plan to cut cancer death rate in half. The New York Times. February 2, 2022.
- Conant EF, Barlow WE, Herschorn SD, et al. Association of digital breast tomosynthesis vs digital mammography with cancer detection and recall rates by age and breast density. JAMA Oncol. 2019;5:635-642. doi: 10.1001 /jamaoncol.2018.7078.
- Tay TK, Tan PH. Liquid biopsy in breast cancer: a focused review. Arch Pathol Lab Med. 2021;145: 678-686. doi: 10.5858/arpa.2019-0559-RA.
- Debald M, Wolgarten M, Walgenbach-Brunagel G, et al. Non-invasive proteomics—thinking about personalized breast cancer screening and treatment. EPMA J. 2010;1:413-420. doi: 10.1007 /s13167-010-0039-9.
Telehealth apps in ObGyn practice
The COVID-19 pandemic has presented increasing demands on health care systems internationally. In addition to redistribution of inpatient health care resources, outpatient care practices evolved, with health care providers offering streamlined access to care to patients via telehealth.
Due to updated insurance practices, physicians now can receive reimbursement via private insurers, Medicare, and Medicaid (as determined by states) for telehealth visits both related and unrelated to COVID-19 care. Increased telehealth use has advantages, including increased health care access, reduced in-clinic wait times, and reduced patient and physician travel time. Within the field of obstetrics and gynecology, clinicians have used telehealth to maintain access to prenatal maternity care while redirecting resources and minimizing the risk of COVID-19 transmission. Additional advantages include provision of care during expanded hours, including evenings and weekends, to increase patient access without increasing the demand on office support staff and the ability to bill for 5- to 10-minute phone counseling encounters.1 Research shows that patients express satisfaction regarding the quality of telehealth care in the setting of prenatal care.2
In February 2020, the American College of Obstetricians and Gynecologists (ACOG) released a Committee Opinion regarding telehealth use in ObGyn, a sign of telehealth’s likely long-standing role within the field.3 Within the statement, ACOG commented on the increasing application of telemedicine in all aspects of obstetrics and gynecology and recommended that physicians become acquainted with new technologies and consider using them in their practice.
There is a large opportunity for development of mobile applications (apps) to further streamline telehealth-based medical care. During the pandemic, the Centers for Medicare and Medicaid Services instituted waivers for telemedicine use on non-HIPAA (Health Insurance Portability and Accountability Act) compliant video communications products, such as Google+ Hangout and Skype. However, HIPAA-compliant video services are preferred, and many virtual apps have released methods for patient communication that meet HIPAA guidelines.1,4 These apps offer services such as phone- and video-based patient visits, appointment scheduling, secure physician-patient messaging, and electronic health record (EHR) documentation.
App recommendations
To identify current mobile apps with clinical use for the ObGyn, we conducted a search of the Apple App Store using the term “telehealth” between December 1, 2021 and January 1, 2022. We limited search results to apps that had at least 1,000 user ratings and to HIPAA-compliant user communication apps. Based on our review, we selected 4 apps to highlight here: Doximity, OhMD, Spruce, and Telehealth by SimplePractice (TABLE). We excluded apps that were advertised as having internal medical clinicians with first patient encounter on-demand through the app or that were associated with a singular insurance company or hospital system.

These apps are largely enabled for iOS and Android mobile devices and are offered at a range of price points for individual physician and practice-scale clinical implementation. Most apps offer secure messaging services between health care practitioners in addition to HIPAA-compliant patient messaging. Some apps offer additional features with the aim to increase patient attendance; these include push notifications, appointment reminders, and an option for automated replies with clinic information. For an additional fee, several apps offer integration to established EHR systems.
An additional tool
The COVID-19 pandemic caused health care systems and individual clinicians to rapidly evolve their practices to maintain patient access to essential health care. Notably, the pandemic led to accelerated implementation of virtual health care services. Telehealth apps likely will become another tool that ObGyns can use to improve the efficiency of their clinical practice and expand patient access to care. ●
- Karram M, Baum N. Telemedicine: a primer for today’s ObGyn. OBG Manag. 2020;32:28-32.
- Marko KI, Ganju N, Krapf JM, et al. A mobile prenatal care app to reduce in-person visits: prospective controlled trial. JMIR Mhealth Uhealth. 2019;7:e10520.
- American College of Obstetricians and Gynecologists. Implementing telehealth in practice: committee opinion no. 798. Obstet Gynecol. 2020;135:e73-e79.
- Karram M, Dooley A, de la Houssaye N, et al. Telemedicine: navigating legal issues. OBG Manag. 2020;32:18-24.
The COVID-19 pandemic has presented increasing demands on health care systems internationally. In addition to redistribution of inpatient health care resources, outpatient care practices evolved, with health care providers offering streamlined access to care to patients via telehealth.
Due to updated insurance practices, physicians now can receive reimbursement via private insurers, Medicare, and Medicaid (as determined by states) for telehealth visits both related and unrelated to COVID-19 care. Increased telehealth use has advantages, including increased health care access, reduced in-clinic wait times, and reduced patient and physician travel time. Within the field of obstetrics and gynecology, clinicians have used telehealth to maintain access to prenatal maternity care while redirecting resources and minimizing the risk of COVID-19 transmission. Additional advantages include provision of care during expanded hours, including evenings and weekends, to increase patient access without increasing the demand on office support staff and the ability to bill for 5- to 10-minute phone counseling encounters.1 Research shows that patients express satisfaction regarding the quality of telehealth care in the setting of prenatal care.2
In February 2020, the American College of Obstetricians and Gynecologists (ACOG) released a Committee Opinion regarding telehealth use in ObGyn, a sign of telehealth’s likely long-standing role within the field.3 Within the statement, ACOG commented on the increasing application of telemedicine in all aspects of obstetrics and gynecology and recommended that physicians become acquainted with new technologies and consider using them in their practice.
There is a large opportunity for development of mobile applications (apps) to further streamline telehealth-based medical care. During the pandemic, the Centers for Medicare and Medicaid Services instituted waivers for telemedicine use on non-HIPAA (Health Insurance Portability and Accountability Act) compliant video communications products, such as Google+ Hangout and Skype. However, HIPAA-compliant video services are preferred, and many virtual apps have released methods for patient communication that meet HIPAA guidelines.1,4 These apps offer services such as phone- and video-based patient visits, appointment scheduling, secure physician-patient messaging, and electronic health record (EHR) documentation.
App recommendations
To identify current mobile apps with clinical use for the ObGyn, we conducted a search of the Apple App Store using the term “telehealth” between December 1, 2021 and January 1, 2022. We limited search results to apps that had at least 1,000 user ratings and to HIPAA-compliant user communication apps. Based on our review, we selected 4 apps to highlight here: Doximity, OhMD, Spruce, and Telehealth by SimplePractice (TABLE). We excluded apps that were advertised as having internal medical clinicians with first patient encounter on-demand through the app or that were associated with a singular insurance company or hospital system.

These apps are largely enabled for iOS and Android mobile devices and are offered at a range of price points for individual physician and practice-scale clinical implementation. Most apps offer secure messaging services between health care practitioners in addition to HIPAA-compliant patient messaging. Some apps offer additional features with the aim to increase patient attendance; these include push notifications, appointment reminders, and an option for automated replies with clinic information. For an additional fee, several apps offer integration to established EHR systems.
An additional tool
The COVID-19 pandemic caused health care systems and individual clinicians to rapidly evolve their practices to maintain patient access to essential health care. Notably, the pandemic led to accelerated implementation of virtual health care services. Telehealth apps likely will become another tool that ObGyns can use to improve the efficiency of their clinical practice and expand patient access to care. ●
The COVID-19 pandemic has presented increasing demands on health care systems internationally. In addition to redistribution of inpatient health care resources, outpatient care practices evolved, with health care providers offering streamlined access to care to patients via telehealth.
Due to updated insurance practices, physicians now can receive reimbursement via private insurers, Medicare, and Medicaid (as determined by states) for telehealth visits both related and unrelated to COVID-19 care. Increased telehealth use has advantages, including increased health care access, reduced in-clinic wait times, and reduced patient and physician travel time. Within the field of obstetrics and gynecology, clinicians have used telehealth to maintain access to prenatal maternity care while redirecting resources and minimizing the risk of COVID-19 transmission. Additional advantages include provision of care during expanded hours, including evenings and weekends, to increase patient access without increasing the demand on office support staff and the ability to bill for 5- to 10-minute phone counseling encounters.1 Research shows that patients express satisfaction regarding the quality of telehealth care in the setting of prenatal care.2
In February 2020, the American College of Obstetricians and Gynecologists (ACOG) released a Committee Opinion regarding telehealth use in ObGyn, a sign of telehealth’s likely long-standing role within the field.3 Within the statement, ACOG commented on the increasing application of telemedicine in all aspects of obstetrics and gynecology and recommended that physicians become acquainted with new technologies and consider using them in their practice.
There is a large opportunity for development of mobile applications (apps) to further streamline telehealth-based medical care. During the pandemic, the Centers for Medicare and Medicaid Services instituted waivers for telemedicine use on non-HIPAA (Health Insurance Portability and Accountability Act) compliant video communications products, such as Google+ Hangout and Skype. However, HIPAA-compliant video services are preferred, and many virtual apps have released methods for patient communication that meet HIPAA guidelines.1,4 These apps offer services such as phone- and video-based patient visits, appointment scheduling, secure physician-patient messaging, and electronic health record (EHR) documentation.
App recommendations
To identify current mobile apps with clinical use for the ObGyn, we conducted a search of the Apple App Store using the term “telehealth” between December 1, 2021 and January 1, 2022. We limited search results to apps that had at least 1,000 user ratings and to HIPAA-compliant user communication apps. Based on our review, we selected 4 apps to highlight here: Doximity, OhMD, Spruce, and Telehealth by SimplePractice (TABLE). We excluded apps that were advertised as having internal medical clinicians with first patient encounter on-demand through the app or that were associated with a singular insurance company or hospital system.

These apps are largely enabled for iOS and Android mobile devices and are offered at a range of price points for individual physician and practice-scale clinical implementation. Most apps offer secure messaging services between health care practitioners in addition to HIPAA-compliant patient messaging. Some apps offer additional features with the aim to increase patient attendance; these include push notifications, appointment reminders, and an option for automated replies with clinic information. For an additional fee, several apps offer integration to established EHR systems.
An additional tool
The COVID-19 pandemic caused health care systems and individual clinicians to rapidly evolve their practices to maintain patient access to essential health care. Notably, the pandemic led to accelerated implementation of virtual health care services. Telehealth apps likely will become another tool that ObGyns can use to improve the efficiency of their clinical practice and expand patient access to care. ●
- Karram M, Baum N. Telemedicine: a primer for today’s ObGyn. OBG Manag. 2020;32:28-32.
- Marko KI, Ganju N, Krapf JM, et al. A mobile prenatal care app to reduce in-person visits: prospective controlled trial. JMIR Mhealth Uhealth. 2019;7:e10520.
- American College of Obstetricians and Gynecologists. Implementing telehealth in practice: committee opinion no. 798. Obstet Gynecol. 2020;135:e73-e79.
- Karram M, Dooley A, de la Houssaye N, et al. Telemedicine: navigating legal issues. OBG Manag. 2020;32:18-24.
- Karram M, Baum N. Telemedicine: a primer for today’s ObGyn. OBG Manag. 2020;32:28-32.
- Marko KI, Ganju N, Krapf JM, et al. A mobile prenatal care app to reduce in-person visits: prospective controlled trial. JMIR Mhealth Uhealth. 2019;7:e10520.
- American College of Obstetricians and Gynecologists. Implementing telehealth in practice: committee opinion no. 798. Obstet Gynecol. 2020;135:e73-e79.
- Karram M, Dooley A, de la Houssaye N, et al. Telemedicine: navigating legal issues. OBG Manag. 2020;32:18-24.
The Pharmacological Management of Constipation
Professor Satish Rao, MD is the J. Harold Harrison, MD, Distinguished University Chair in Gastroenterology. Dr. Rao is also founding Director of the Digestive Health Center and Clinical Research Center, and tenured Director and Professor of Medicine. His research interests in Neurogastroenterology/Motility have focused on gaining mechanistic insights, developing novel diagnostic tools and pioneering innovative treatments for constipation, dyssynergic defecation, fecal incontinence, IBS, food intolerance, gas and bloating and small intestinal bacterial and fungal overgrowth (SIBO/SIFO) and visceral pain. His latest invention translumbosacral neuromodulation therapy (TNT) is revolutionizing treatment for fecal incontinence.
As a gastroenterologist, with a focus on research that includes pathophysiological treatment of IBS and constipation as the primary objective, how prevalent is it in your current practice?
Dr. Rao: It is a pleasure to discuss a topic very close to my heart. It is a very important but often neglected topic, and very many times people go to pharmacies, over-the-counter, or their grandmothers, seeking treatment for constipation, whereas, with all the advances today, they should be coming to us, gastroenterologists, as the primary source for really managing this problem.
Constipation is very common. It all depends, to some extent, on how we define it. But if we define it based on some more popular criteria, such as those supported by the Rome Foundation criteria, the global prevalence, is between 10% to 15%. As you can see millions of Americans suffer with this problem, and almost all AGA members would have seen hundreds of these patients in the course of their practice every year. So, it is highly prevalent.
The term constipation is misunderstood by many people. Different people have different names, different people have different definitions and different criteria. For years, most physicians and most textbooks equated constipation to infrequent bowel movements. That logic has changed dramatically in the last 10 to 15 years, where we now recognize constipation as not only infrequent bowel movement but, more commonly, difficulty with bowel movement. This difficulty with bowel movement has been the missing link as we were all focused on infrequent bowel movement. We now recognize that constipation means one of six things.
What are those six things that tell us it’s constipation? One, there’s excessive straining to have a bowel movement; two after having had a bowel movement, you're left with a feeling of incomplete evacuation; three, the stools are hard and difficult to pass. We have a very famous scale, called the Bristol Stool Form Scale. If anybody takes the time to look at the scale, if your stool form happens to be 1, 2, or 3, then you're more likely to be constipated; four, a patient has to use digital maneuvers or some kind of support to try and evacuate stool; five, a patient reports a sensation of blockage at the time of bowel movement repeatedly, and at least with 25% of bowel movements; and six, stool frequency of less than three bowel movements per week.
In order to diagnose constipation, if a patient has any two of these symptoms, for 25% of bowel movements over a period of three to six months, then that individual should be considered as having chronic constipation.
When is pharmacological management of constipation appropriate? And what diagnostic approach do you usually take to determine treatment?
Dr. Rao: I usually take a very detailed history from these patients. One of the things we have recognized, recently, is how inadequate our history has become, not necessarily from a lack of asking questions, but it seems to be a multifactorial process. We tested this in a prospective study showing that only 30% of the time were patients history correct in letting us know why they came to the clinic. The same patient who answered a questionnaire about their symptoms, when they keep a diary for one week, there's only 30% of the time there is concordance; 70% of the time the story is different.
Hence, the first step really is to get an accurate story about your patient with constipation. Fortunately, there are some digital apps that are available, a constipation stool diary app, and there is a MyGiHealth app, et cetera. People can use these apps or they can keep a paper diary. The next step would be to determine what may be mechanistically going wrong. These two steps will guide your management approach.
We have some simple tests that we can do. One is called a colon transit study, where we measure the speed at which stool goes through the colon. And we have two tests that are commonly used. One is called a Sitz marker test. The patient swallows the capsule which has 24 plastic rings. And then they take it on day one. They get an X-ray on day six, which is 120 hours later, and you count how many of those markers are left behind with the X-ray. If there are more than five, it's abnormal. It says they have slow transit. In other words, lazy colon. On the other hand, if they pass most of the markers, then they don't have a lazy colon.
The second test we often do is a wireless motility capsule test, which is again a capsule they swallow. They wear a recorder for five days. And then that measures the speed at which the capsule goes through the colon. A very simple way of measuring colon speed as well. These tests tell us if you have a lazy colon.
Another test that we do is anorectal manometry. We do the test because 40%-50% of patients have pelvic floor dysfunction. And that gives us important insights mechanistically whether they have rectal hypersensitivity or whether they have a problem with evacuating stool. If they have rectal hypersensitivity and they're complaining of constipation, then it equates to irritable bowel syndrome with constipation. That's how we pick up that category of constipation.
The other category I mentioned earlier, lazy colon, is slow transit constipation. And the third category we often see is called dyssynergic defecation, where they are unable to evacuate stool in a timely, orderly fashion. And that is the third group of constipation. Use these tests to help you diagnose which kind of constipation a patient has. That will then guide towards appropriate management.
Is there a higher frequency in the need for pharmacological management of constipation? And is it in any particular patient, for instance, such as older adults, the elderly, or those that may be critically ill, or any other demographic?
Dr. Rao: In terms of managing the constipation itself, many times patients will go to the drugstore or talk to a pharmacist and start taking some over-the-counter preparations.
Until recently, there hasn’t been any good study to tell us which one works, which preparation works. But we recently published a systematic review of which I had the pleasure of serving as the first author, where we looked at the 30-year data on over-the-counter treatments. We found that there is very good evidence to support polyethylene glycol, PEG, as grade A, recommendation.
We also found good evidence for senna, and for magnesium. These three compounds had good evidence. Whereas, for other over-the-counter preparations, such as fiber supplements, there are some fruit-based supplements that are now available, lactulose and so on, the evidence was second grade.
With regards to prescriptions, a drug that is approved is linaclotide, which is approved at a dose of 145 micrograms a day in a capsule form for treatment of chronic constipation. It's also approved for IBS with constipation at a higher dose of 290 micrograms a day. We also have plecanatide 3mg tablets. Both linaclotide and plecanatide, are guanylate cyclase C receptor agonists. They activate secretion through the guanylate cyclase pathway in the gut. And then, by inducing secretion, they improve constipation. Both drugs are approved for IBS-C and chronic constipation.
Then we have lubiprostone, which is a chloride channel 2 activator drug that is approved at a dose of 24 micrograms twice a day with food for treatment of chronic constipation, and 8 micrograms twice a day for treatment of IBS with constipation.
The most recent drug that was approved is called prucalopride. And this is approved at 1 to 2 milligrams a day. And this is a serotonin compound, which speeds up the gut activity including the, stomach, small bowel, and the colon. And by speeding up gut activity, it improves constipation.
Regarding particular demographics, I find that older patients tend to be a little bit more sensitive to some drugs. Some older adults are more refractory to standard compounds, which makes their management a little bit more challenging. We have to really titrate the dose very carefully.
The critically ill group is a little bit more challenging. Often, it's acute constipation or they're in ICU settings and there the management becomes a little bit more complex. One component of that critically ill group-- and of course, you can also see them in outpatient practice-- is the opioid-induced constipation, which is a category in its own right and has been recognized by the FDA. Unfortunately, one of the major side effects of opioid is constipation. It takes a toll in the gut.
Fortunately, we have a new set of drugs, called PAMORAs, or peripherally acting mu-opioid receptor antagonists. These drugs, when you take them orally, will neutralize the effect of opioids in the gut, and thereby relieve constipation without affecting the analgesic effect of opioids. Examples of these class of drugs are methylnaltrexone, naloxegol, naldemedine, etc. These are all FDA approved now for treatment of opioid-induced constipation, which is part of your critical ill or hospitalized patients, and sometimes they are really outpatients as well.
Are there risk factors?
Dr. Rao: There are several risk factors for constipation. They include, for example, elderly, particularly people who are not very mobile for various reasons, people-- but I think the biggest risk factor that I would like to mention, emphasize, is drugs. There are many, many, many drugs that cause constipation. Opioids, as I just mentioned . Also a number of anti-hypertensive drugs. Calcium channel blockers, for example, are constipating.
Now, iron, heavy metals, and calcium, are very constipating. So are anticholinergics, and antidepressants. Many antidepressants, particularly the tricyclic class, are very constipating.
My first message to my colleagues is, when a patient is presenting to you in the clinic, the first thing I teach my students, residents, fellows, is to look at the drug list. Think about that drug as a mischief for constipation. If it is feasible and appropriate, remove the drug, or substitute the drug, as you are looking for other reasons for constipation.
Another important risk factor worth mentioning is acute constipation. Constipation in a 70-year-old person, suddenly over the last six weeks, is serious and may raise suspicion for cancer in the colon, and the need for investigation. If patients suddenly develop constipation like that, then, it is important to make sure that there is nothing blocking the colon, that is creating constipation.
One other important group is pregnancy. I did forget to mention that. Interestingly, between a quarter to a third of woman, otherwise healthy women, have never had any symptoms, however, during pregnancy they do become constipated. That's because of hormonal changes, particularly the rush of progesterone which is happening in their body. And so, they will have to be managed during their stage, and very many times they will come back to their normal lifestyle afterwards.
When it comes to the data on current and emerging treatments, what do you see in the future of pharmacological management of constipation?
Dr. Rao: I think one of the critical things in the pharmacologic management of constipation is to recognize that constipation is rarely a one-mechanism disorder. And it may be in a particular individual, but how do we know? For example, how do we know that their gut is not producing enough serotonin? And that is why they are serotonin depleted in the gut and we need to supplement them with a serotonin product, to just give you an example. We don't know that. We don't have their genetic makeup, and so on.
What I'm getting at is, constipation is a heterogenic problem, and there are multiple mechanisms that lead to it. Therefore, our current armamentarium has significantly improved in the last decade.
The first decade of the new Millennium saw significant new drugs that were introduced. The second decade has seen even more. But I think there are other compounds that are now coming up. There are sodium-hydrogen ion pump blocking drugs , and there are other mechanistic drugs . There's another drug which is available in Japan, not yet in the US, called elobixibat, which actually blocks bile acid. Normally, 95% of the bile is reabsorbed in the colon. But if you block bile acid reabsorption and allow some bile to spill into the colon, your own bile can become a laxative in the colon.
Another important approach has been through a capsule technology, called Vibrant capsule. . I'm part of an investigative group is investigating this drug. The phase III data is not yet available but has been submitted, You take a capsule once a day, and this gently agitates the colon, and thereby stimulating the colon muscles to move the stool, and then you evacuate. So, it is not a pharmacologic, but it is a form of a capsule device treatment .
These are some emerging treatments just around the corner. There is kind of a belt device that you can wear around the belly, which passes a small amount of electric current in a sequential manner, to stimulate peristalsis, called electrical interference therapy.
If pharmacological management is not an option, then what's next?
Dr. Rao: Pharmacologic management works for about 50% of patients. But it doesn't work in everybody. It's not because the drug itself is not working or has side effects, instead, the issue is that the problem is not likely to be fixed by pharmacologic management.
About 40% of patients have a pelvic floor dysfunction called dyssynergic defecation. These folks, unbeknownst to them, have learned a new process of pooping, where they are blocking their own pooping action. They're not doing it deliberately. They're totally unaware. A third of them have this problem right from childhood. 2/3 acquire this problem in adulthood. Needless to say, this problem affects 40% of patients.
So yes, you can give them medications, and that will temporarily help them. But because they cannot evacuate this stool, it will never help them permanently. These individuals are best helped by a behavioral treatment called biofeedback therapy. Hopefully in the future, home-based biofeedback tools can become available, and that can really make this treatment more widely available to the public.
Is there anything else you'd like to add before we conclude?
Dr. Rao: My most important message to my colleagues is that constipation is a very common problem. Please take time to spend with your patients. Please use an app or a diary to record the symptoms. If at all possible, perform a digital rectal examination on all your constipated patients to identify the pelvic floor dysfunction group of patients. Manage them appropriately with over the counter drugs or particularly the FDA-approved pharmacotherapies. And that will work in a majority.
But when drugs don’t work or patients have predominant symptoms of difficult defecation, put on your thinking hat. Don't give up on your patients. Instead of sending them to a surgeon, which many times we rush to do, try and see what other things you can do including manometric testing and biofeedback therapy.
Professor Satish Rao, MD is the J. Harold Harrison, MD, Distinguished University Chair in Gastroenterology. Dr. Rao is also founding Director of the Digestive Health Center and Clinical Research Center, and tenured Director and Professor of Medicine. His research interests in Neurogastroenterology/Motility have focused on gaining mechanistic insights, developing novel diagnostic tools and pioneering innovative treatments for constipation, dyssynergic defecation, fecal incontinence, IBS, food intolerance, gas and bloating and small intestinal bacterial and fungal overgrowth (SIBO/SIFO) and visceral pain. His latest invention translumbosacral neuromodulation therapy (TNT) is revolutionizing treatment for fecal incontinence.
As a gastroenterologist, with a focus on research that includes pathophysiological treatment of IBS and constipation as the primary objective, how prevalent is it in your current practice?
Dr. Rao: It is a pleasure to discuss a topic very close to my heart. It is a very important but often neglected topic, and very many times people go to pharmacies, over-the-counter, or their grandmothers, seeking treatment for constipation, whereas, with all the advances today, they should be coming to us, gastroenterologists, as the primary source for really managing this problem.
Constipation is very common. It all depends, to some extent, on how we define it. But if we define it based on some more popular criteria, such as those supported by the Rome Foundation criteria, the global prevalence, is between 10% to 15%. As you can see millions of Americans suffer with this problem, and almost all AGA members would have seen hundreds of these patients in the course of their practice every year. So, it is highly prevalent.
The term constipation is misunderstood by many people. Different people have different names, different people have different definitions and different criteria. For years, most physicians and most textbooks equated constipation to infrequent bowel movements. That logic has changed dramatically in the last 10 to 15 years, where we now recognize constipation as not only infrequent bowel movement but, more commonly, difficulty with bowel movement. This difficulty with bowel movement has been the missing link as we were all focused on infrequent bowel movement. We now recognize that constipation means one of six things.
What are those six things that tell us it’s constipation? One, there’s excessive straining to have a bowel movement; two after having had a bowel movement, you're left with a feeling of incomplete evacuation; three, the stools are hard and difficult to pass. We have a very famous scale, called the Bristol Stool Form Scale. If anybody takes the time to look at the scale, if your stool form happens to be 1, 2, or 3, then you're more likely to be constipated; four, a patient has to use digital maneuvers or some kind of support to try and evacuate stool; five, a patient reports a sensation of blockage at the time of bowel movement repeatedly, and at least with 25% of bowel movements; and six, stool frequency of less than three bowel movements per week.
In order to diagnose constipation, if a patient has any two of these symptoms, for 25% of bowel movements over a period of three to six months, then that individual should be considered as having chronic constipation.
When is pharmacological management of constipation appropriate? And what diagnostic approach do you usually take to determine treatment?
Dr. Rao: I usually take a very detailed history from these patients. One of the things we have recognized, recently, is how inadequate our history has become, not necessarily from a lack of asking questions, but it seems to be a multifactorial process. We tested this in a prospective study showing that only 30% of the time were patients history correct in letting us know why they came to the clinic. The same patient who answered a questionnaire about their symptoms, when they keep a diary for one week, there's only 30% of the time there is concordance; 70% of the time the story is different.
Hence, the first step really is to get an accurate story about your patient with constipation. Fortunately, there are some digital apps that are available, a constipation stool diary app, and there is a MyGiHealth app, et cetera. People can use these apps or they can keep a paper diary. The next step would be to determine what may be mechanistically going wrong. These two steps will guide your management approach.
We have some simple tests that we can do. One is called a colon transit study, where we measure the speed at which stool goes through the colon. And we have two tests that are commonly used. One is called a Sitz marker test. The patient swallows the capsule which has 24 plastic rings. And then they take it on day one. They get an X-ray on day six, which is 120 hours later, and you count how many of those markers are left behind with the X-ray. If there are more than five, it's abnormal. It says they have slow transit. In other words, lazy colon. On the other hand, if they pass most of the markers, then they don't have a lazy colon.
The second test we often do is a wireless motility capsule test, which is again a capsule they swallow. They wear a recorder for five days. And then that measures the speed at which the capsule goes through the colon. A very simple way of measuring colon speed as well. These tests tell us if you have a lazy colon.
Another test that we do is anorectal manometry. We do the test because 40%-50% of patients have pelvic floor dysfunction. And that gives us important insights mechanistically whether they have rectal hypersensitivity or whether they have a problem with evacuating stool. If they have rectal hypersensitivity and they're complaining of constipation, then it equates to irritable bowel syndrome with constipation. That's how we pick up that category of constipation.
The other category I mentioned earlier, lazy colon, is slow transit constipation. And the third category we often see is called dyssynergic defecation, where they are unable to evacuate stool in a timely, orderly fashion. And that is the third group of constipation. Use these tests to help you diagnose which kind of constipation a patient has. That will then guide towards appropriate management.
Is there a higher frequency in the need for pharmacological management of constipation? And is it in any particular patient, for instance, such as older adults, the elderly, or those that may be critically ill, or any other demographic?
Dr. Rao: In terms of managing the constipation itself, many times patients will go to the drugstore or talk to a pharmacist and start taking some over-the-counter preparations.
Until recently, there hasn’t been any good study to tell us which one works, which preparation works. But we recently published a systematic review of which I had the pleasure of serving as the first author, where we looked at the 30-year data on over-the-counter treatments. We found that there is very good evidence to support polyethylene glycol, PEG, as grade A, recommendation.
We also found good evidence for senna, and for magnesium. These three compounds had good evidence. Whereas, for other over-the-counter preparations, such as fiber supplements, there are some fruit-based supplements that are now available, lactulose and so on, the evidence was second grade.
With regards to prescriptions, a drug that is approved is linaclotide, which is approved at a dose of 145 micrograms a day in a capsule form for treatment of chronic constipation. It's also approved for IBS with constipation at a higher dose of 290 micrograms a day. We also have plecanatide 3mg tablets. Both linaclotide and plecanatide, are guanylate cyclase C receptor agonists. They activate secretion through the guanylate cyclase pathway in the gut. And then, by inducing secretion, they improve constipation. Both drugs are approved for IBS-C and chronic constipation.
Then we have lubiprostone, which is a chloride channel 2 activator drug that is approved at a dose of 24 micrograms twice a day with food for treatment of chronic constipation, and 8 micrograms twice a day for treatment of IBS with constipation.
The most recent drug that was approved is called prucalopride. And this is approved at 1 to 2 milligrams a day. And this is a serotonin compound, which speeds up the gut activity including the, stomach, small bowel, and the colon. And by speeding up gut activity, it improves constipation.
Regarding particular demographics, I find that older patients tend to be a little bit more sensitive to some drugs. Some older adults are more refractory to standard compounds, which makes their management a little bit more challenging. We have to really titrate the dose very carefully.
The critically ill group is a little bit more challenging. Often, it's acute constipation or they're in ICU settings and there the management becomes a little bit more complex. One component of that critically ill group-- and of course, you can also see them in outpatient practice-- is the opioid-induced constipation, which is a category in its own right and has been recognized by the FDA. Unfortunately, one of the major side effects of opioid is constipation. It takes a toll in the gut.
Fortunately, we have a new set of drugs, called PAMORAs, or peripherally acting mu-opioid receptor antagonists. These drugs, when you take them orally, will neutralize the effect of opioids in the gut, and thereby relieve constipation without affecting the analgesic effect of opioids. Examples of these class of drugs are methylnaltrexone, naloxegol, naldemedine, etc. These are all FDA approved now for treatment of opioid-induced constipation, which is part of your critical ill or hospitalized patients, and sometimes they are really outpatients as well.
Are there risk factors?
Dr. Rao: There are several risk factors for constipation. They include, for example, elderly, particularly people who are not very mobile for various reasons, people-- but I think the biggest risk factor that I would like to mention, emphasize, is drugs. There are many, many, many drugs that cause constipation. Opioids, as I just mentioned . Also a number of anti-hypertensive drugs. Calcium channel blockers, for example, are constipating.
Now, iron, heavy metals, and calcium, are very constipating. So are anticholinergics, and antidepressants. Many antidepressants, particularly the tricyclic class, are very constipating.
My first message to my colleagues is, when a patient is presenting to you in the clinic, the first thing I teach my students, residents, fellows, is to look at the drug list. Think about that drug as a mischief for constipation. If it is feasible and appropriate, remove the drug, or substitute the drug, as you are looking for other reasons for constipation.
Another important risk factor worth mentioning is acute constipation. Constipation in a 70-year-old person, suddenly over the last six weeks, is serious and may raise suspicion for cancer in the colon, and the need for investigation. If patients suddenly develop constipation like that, then, it is important to make sure that there is nothing blocking the colon, that is creating constipation.
One other important group is pregnancy. I did forget to mention that. Interestingly, between a quarter to a third of woman, otherwise healthy women, have never had any symptoms, however, during pregnancy they do become constipated. That's because of hormonal changes, particularly the rush of progesterone which is happening in their body. And so, they will have to be managed during their stage, and very many times they will come back to their normal lifestyle afterwards.
When it comes to the data on current and emerging treatments, what do you see in the future of pharmacological management of constipation?
Dr. Rao: I think one of the critical things in the pharmacologic management of constipation is to recognize that constipation is rarely a one-mechanism disorder. And it may be in a particular individual, but how do we know? For example, how do we know that their gut is not producing enough serotonin? And that is why they are serotonin depleted in the gut and we need to supplement them with a serotonin product, to just give you an example. We don't know that. We don't have their genetic makeup, and so on.
What I'm getting at is, constipation is a heterogenic problem, and there are multiple mechanisms that lead to it. Therefore, our current armamentarium has significantly improved in the last decade.
The first decade of the new Millennium saw significant new drugs that were introduced. The second decade has seen even more. But I think there are other compounds that are now coming up. There are sodium-hydrogen ion pump blocking drugs , and there are other mechanistic drugs . There's another drug which is available in Japan, not yet in the US, called elobixibat, which actually blocks bile acid. Normally, 95% of the bile is reabsorbed in the colon. But if you block bile acid reabsorption and allow some bile to spill into the colon, your own bile can become a laxative in the colon.
Another important approach has been through a capsule technology, called Vibrant capsule. . I'm part of an investigative group is investigating this drug. The phase III data is not yet available but has been submitted, You take a capsule once a day, and this gently agitates the colon, and thereby stimulating the colon muscles to move the stool, and then you evacuate. So, it is not a pharmacologic, but it is a form of a capsule device treatment .
These are some emerging treatments just around the corner. There is kind of a belt device that you can wear around the belly, which passes a small amount of electric current in a sequential manner, to stimulate peristalsis, called electrical interference therapy.
If pharmacological management is not an option, then what's next?
Dr. Rao: Pharmacologic management works for about 50% of patients. But it doesn't work in everybody. It's not because the drug itself is not working or has side effects, instead, the issue is that the problem is not likely to be fixed by pharmacologic management.
About 40% of patients have a pelvic floor dysfunction called dyssynergic defecation. These folks, unbeknownst to them, have learned a new process of pooping, where they are blocking their own pooping action. They're not doing it deliberately. They're totally unaware. A third of them have this problem right from childhood. 2/3 acquire this problem in adulthood. Needless to say, this problem affects 40% of patients.
So yes, you can give them medications, and that will temporarily help them. But because they cannot evacuate this stool, it will never help them permanently. These individuals are best helped by a behavioral treatment called biofeedback therapy. Hopefully in the future, home-based biofeedback tools can become available, and that can really make this treatment more widely available to the public.
Is there anything else you'd like to add before we conclude?
Dr. Rao: My most important message to my colleagues is that constipation is a very common problem. Please take time to spend with your patients. Please use an app or a diary to record the symptoms. If at all possible, perform a digital rectal examination on all your constipated patients to identify the pelvic floor dysfunction group of patients. Manage them appropriately with over the counter drugs or particularly the FDA-approved pharmacotherapies. And that will work in a majority.
But when drugs don’t work or patients have predominant symptoms of difficult defecation, put on your thinking hat. Don't give up on your patients. Instead of sending them to a surgeon, which many times we rush to do, try and see what other things you can do including manometric testing and biofeedback therapy.
Professor Satish Rao, MD is the J. Harold Harrison, MD, Distinguished University Chair in Gastroenterology. Dr. Rao is also founding Director of the Digestive Health Center and Clinical Research Center, and tenured Director and Professor of Medicine. His research interests in Neurogastroenterology/Motility have focused on gaining mechanistic insights, developing novel diagnostic tools and pioneering innovative treatments for constipation, dyssynergic defecation, fecal incontinence, IBS, food intolerance, gas and bloating and small intestinal bacterial and fungal overgrowth (SIBO/SIFO) and visceral pain. His latest invention translumbosacral neuromodulation therapy (TNT) is revolutionizing treatment for fecal incontinence.
As a gastroenterologist, with a focus on research that includes pathophysiological treatment of IBS and constipation as the primary objective, how prevalent is it in your current practice?
Dr. Rao: It is a pleasure to discuss a topic very close to my heart. It is a very important but often neglected topic, and very many times people go to pharmacies, over-the-counter, or their grandmothers, seeking treatment for constipation, whereas, with all the advances today, they should be coming to us, gastroenterologists, as the primary source for really managing this problem.
Constipation is very common. It all depends, to some extent, on how we define it. But if we define it based on some more popular criteria, such as those supported by the Rome Foundation criteria, the global prevalence, is between 10% to 15%. As you can see millions of Americans suffer with this problem, and almost all AGA members would have seen hundreds of these patients in the course of their practice every year. So, it is highly prevalent.
The term constipation is misunderstood by many people. Different people have different names, different people have different definitions and different criteria. For years, most physicians and most textbooks equated constipation to infrequent bowel movements. That logic has changed dramatically in the last 10 to 15 years, where we now recognize constipation as not only infrequent bowel movement but, more commonly, difficulty with bowel movement. This difficulty with bowel movement has been the missing link as we were all focused on infrequent bowel movement. We now recognize that constipation means one of six things.
What are those six things that tell us it’s constipation? One, there’s excessive straining to have a bowel movement; two after having had a bowel movement, you're left with a feeling of incomplete evacuation; three, the stools are hard and difficult to pass. We have a very famous scale, called the Bristol Stool Form Scale. If anybody takes the time to look at the scale, if your stool form happens to be 1, 2, or 3, then you're more likely to be constipated; four, a patient has to use digital maneuvers or some kind of support to try and evacuate stool; five, a patient reports a sensation of blockage at the time of bowel movement repeatedly, and at least with 25% of bowel movements; and six, stool frequency of less than three bowel movements per week.
In order to diagnose constipation, if a patient has any two of these symptoms, for 25% of bowel movements over a period of three to six months, then that individual should be considered as having chronic constipation.
When is pharmacological management of constipation appropriate? And what diagnostic approach do you usually take to determine treatment?
Dr. Rao: I usually take a very detailed history from these patients. One of the things we have recognized, recently, is how inadequate our history has become, not necessarily from a lack of asking questions, but it seems to be a multifactorial process. We tested this in a prospective study showing that only 30% of the time were patients history correct in letting us know why they came to the clinic. The same patient who answered a questionnaire about their symptoms, when they keep a diary for one week, there's only 30% of the time there is concordance; 70% of the time the story is different.
Hence, the first step really is to get an accurate story about your patient with constipation. Fortunately, there are some digital apps that are available, a constipation stool diary app, and there is a MyGiHealth app, et cetera. People can use these apps or they can keep a paper diary. The next step would be to determine what may be mechanistically going wrong. These two steps will guide your management approach.
We have some simple tests that we can do. One is called a colon transit study, where we measure the speed at which stool goes through the colon. And we have two tests that are commonly used. One is called a Sitz marker test. The patient swallows the capsule which has 24 plastic rings. And then they take it on day one. They get an X-ray on day six, which is 120 hours later, and you count how many of those markers are left behind with the X-ray. If there are more than five, it's abnormal. It says they have slow transit. In other words, lazy colon. On the other hand, if they pass most of the markers, then they don't have a lazy colon.
The second test we often do is a wireless motility capsule test, which is again a capsule they swallow. They wear a recorder for five days. And then that measures the speed at which the capsule goes through the colon. A very simple way of measuring colon speed as well. These tests tell us if you have a lazy colon.
Another test that we do is anorectal manometry. We do the test because 40%-50% of patients have pelvic floor dysfunction. And that gives us important insights mechanistically whether they have rectal hypersensitivity or whether they have a problem with evacuating stool. If they have rectal hypersensitivity and they're complaining of constipation, then it equates to irritable bowel syndrome with constipation. That's how we pick up that category of constipation.
The other category I mentioned earlier, lazy colon, is slow transit constipation. And the third category we often see is called dyssynergic defecation, where they are unable to evacuate stool in a timely, orderly fashion. And that is the third group of constipation. Use these tests to help you diagnose which kind of constipation a patient has. That will then guide towards appropriate management.
Is there a higher frequency in the need for pharmacological management of constipation? And is it in any particular patient, for instance, such as older adults, the elderly, or those that may be critically ill, or any other demographic?
Dr. Rao: In terms of managing the constipation itself, many times patients will go to the drugstore or talk to a pharmacist and start taking some over-the-counter preparations.
Until recently, there hasn’t been any good study to tell us which one works, which preparation works. But we recently published a systematic review of which I had the pleasure of serving as the first author, where we looked at the 30-year data on over-the-counter treatments. We found that there is very good evidence to support polyethylene glycol, PEG, as grade A, recommendation.
We also found good evidence for senna, and for magnesium. These three compounds had good evidence. Whereas, for other over-the-counter preparations, such as fiber supplements, there are some fruit-based supplements that are now available, lactulose and so on, the evidence was second grade.
With regards to prescriptions, a drug that is approved is linaclotide, which is approved at a dose of 145 micrograms a day in a capsule form for treatment of chronic constipation. It's also approved for IBS with constipation at a higher dose of 290 micrograms a day. We also have plecanatide 3mg tablets. Both linaclotide and plecanatide, are guanylate cyclase C receptor agonists. They activate secretion through the guanylate cyclase pathway in the gut. And then, by inducing secretion, they improve constipation. Both drugs are approved for IBS-C and chronic constipation.
Then we have lubiprostone, which is a chloride channel 2 activator drug that is approved at a dose of 24 micrograms twice a day with food for treatment of chronic constipation, and 8 micrograms twice a day for treatment of IBS with constipation.
The most recent drug that was approved is called prucalopride. And this is approved at 1 to 2 milligrams a day. And this is a serotonin compound, which speeds up the gut activity including the, stomach, small bowel, and the colon. And by speeding up gut activity, it improves constipation.
Regarding particular demographics, I find that older patients tend to be a little bit more sensitive to some drugs. Some older adults are more refractory to standard compounds, which makes their management a little bit more challenging. We have to really titrate the dose very carefully.
The critically ill group is a little bit more challenging. Often, it's acute constipation or they're in ICU settings and there the management becomes a little bit more complex. One component of that critically ill group-- and of course, you can also see them in outpatient practice-- is the opioid-induced constipation, which is a category in its own right and has been recognized by the FDA. Unfortunately, one of the major side effects of opioid is constipation. It takes a toll in the gut.
Fortunately, we have a new set of drugs, called PAMORAs, or peripherally acting mu-opioid receptor antagonists. These drugs, when you take them orally, will neutralize the effect of opioids in the gut, and thereby relieve constipation without affecting the analgesic effect of opioids. Examples of these class of drugs are methylnaltrexone, naloxegol, naldemedine, etc. These are all FDA approved now for treatment of opioid-induced constipation, which is part of your critical ill or hospitalized patients, and sometimes they are really outpatients as well.
Are there risk factors?
Dr. Rao: There are several risk factors for constipation. They include, for example, elderly, particularly people who are not very mobile for various reasons, people-- but I think the biggest risk factor that I would like to mention, emphasize, is drugs. There are many, many, many drugs that cause constipation. Opioids, as I just mentioned . Also a number of anti-hypertensive drugs. Calcium channel blockers, for example, are constipating.
Now, iron, heavy metals, and calcium, are very constipating. So are anticholinergics, and antidepressants. Many antidepressants, particularly the tricyclic class, are very constipating.
My first message to my colleagues is, when a patient is presenting to you in the clinic, the first thing I teach my students, residents, fellows, is to look at the drug list. Think about that drug as a mischief for constipation. If it is feasible and appropriate, remove the drug, or substitute the drug, as you are looking for other reasons for constipation.
Another important risk factor worth mentioning is acute constipation. Constipation in a 70-year-old person, suddenly over the last six weeks, is serious and may raise suspicion for cancer in the colon, and the need for investigation. If patients suddenly develop constipation like that, then, it is important to make sure that there is nothing blocking the colon, that is creating constipation.
One other important group is pregnancy. I did forget to mention that. Interestingly, between a quarter to a third of woman, otherwise healthy women, have never had any symptoms, however, during pregnancy they do become constipated. That's because of hormonal changes, particularly the rush of progesterone which is happening in their body. And so, they will have to be managed during their stage, and very many times they will come back to their normal lifestyle afterwards.
When it comes to the data on current and emerging treatments, what do you see in the future of pharmacological management of constipation?
Dr. Rao: I think one of the critical things in the pharmacologic management of constipation is to recognize that constipation is rarely a one-mechanism disorder. And it may be in a particular individual, but how do we know? For example, how do we know that their gut is not producing enough serotonin? And that is why they are serotonin depleted in the gut and we need to supplement them with a serotonin product, to just give you an example. We don't know that. We don't have their genetic makeup, and so on.
What I'm getting at is, constipation is a heterogenic problem, and there are multiple mechanisms that lead to it. Therefore, our current armamentarium has significantly improved in the last decade.
The first decade of the new Millennium saw significant new drugs that were introduced. The second decade has seen even more. But I think there are other compounds that are now coming up. There are sodium-hydrogen ion pump blocking drugs , and there are other mechanistic drugs . There's another drug which is available in Japan, not yet in the US, called elobixibat, which actually blocks bile acid. Normally, 95% of the bile is reabsorbed in the colon. But if you block bile acid reabsorption and allow some bile to spill into the colon, your own bile can become a laxative in the colon.
Another important approach has been through a capsule technology, called Vibrant capsule. . I'm part of an investigative group is investigating this drug. The phase III data is not yet available but has been submitted, You take a capsule once a day, and this gently agitates the colon, and thereby stimulating the colon muscles to move the stool, and then you evacuate. So, it is not a pharmacologic, but it is a form of a capsule device treatment .
These are some emerging treatments just around the corner. There is kind of a belt device that you can wear around the belly, which passes a small amount of electric current in a sequential manner, to stimulate peristalsis, called electrical interference therapy.
If pharmacological management is not an option, then what's next?
Dr. Rao: Pharmacologic management works for about 50% of patients. But it doesn't work in everybody. It's not because the drug itself is not working or has side effects, instead, the issue is that the problem is not likely to be fixed by pharmacologic management.
About 40% of patients have a pelvic floor dysfunction called dyssynergic defecation. These folks, unbeknownst to them, have learned a new process of pooping, where they are blocking their own pooping action. They're not doing it deliberately. They're totally unaware. A third of them have this problem right from childhood. 2/3 acquire this problem in adulthood. Needless to say, this problem affects 40% of patients.
So yes, you can give them medications, and that will temporarily help them. But because they cannot evacuate this stool, it will never help them permanently. These individuals are best helped by a behavioral treatment called biofeedback therapy. Hopefully in the future, home-based biofeedback tools can become available, and that can really make this treatment more widely available to the public.
Is there anything else you'd like to add before we conclude?
Dr. Rao: My most important message to my colleagues is that constipation is a very common problem. Please take time to spend with your patients. Please use an app or a diary to record the symptoms. If at all possible, perform a digital rectal examination on all your constipated patients to identify the pelvic floor dysfunction group of patients. Manage them appropriately with over the counter drugs or particularly the FDA-approved pharmacotherapies. And that will work in a majority.
But when drugs don’t work or patients have predominant symptoms of difficult defecation, put on your thinking hat. Don't give up on your patients. Instead of sending them to a surgeon, which many times we rush to do, try and see what other things you can do including manometric testing and biofeedback therapy.
Neurologic Care Isn’t Reducible to a Code
Neurology, once considered a “diagnose and adios” specialty, is gaining newfound, scientific respect. Our vastly improved understanding of neurologic pathophysiology has led to many Food and Drug Administration–approved medications that can specifically enhance treatment outcomes. Medications for migraine, multiple sclerosis, epilepsy, and other chronic neurological diseases have been extended and modernized; for millions of patients, these medicines fulfill their long-awaited needs.
What would Dr Osler have said if he witnessed today’s definition of the practice of medicine? As singular as our patients and their disorders are, the delivery of care is anything but. The processes in the delivery of this care have created many unforeseen twists and turns, thanks to the electronic health record (EHR), the resource-based relative value scale (RBRVS), evaluation and management (E&M) coding, and private health insurance (PHI).
From a neurologist’s perspective, I will elaborate upon these changes that have affected our day-to-day neurology practices. I have practiced general neurology and headache medicine both in private and academic practices, evaluating and treating thousands of inpatients and outpatients in urban and rural healthcare facilities since 1986.
The EHR
Despite herculean, lofty, and sustained efforts by the medical business world to promote EHR adoption worldwide, goals remain unmet. Intended to improve the quality of care and patient outcomes, reduce medical errors, and crystalize communications among providers and with patients, it is instead associated with physician burnout (B), lack of usability (U) and interoperability (I), has likability (L) issues, and provides no productive physician direction (D) – there is an enormous need to BUILD it better.
In my own practice, it is inevitable that I will use my EHR laptop with an unknowing patient. If so, I try to make her feel comfortable in its presence as I strive to stay intent on our discussion. Yet I invariably split my concentration between machine and patient. The machine often gets my full attention, with its confusing and unnecessary medical record notes, tech glitches and screen interruptions, let alone its complicated web of tabs, buttons, links, and obscure prompts. As for fulfilling CMS’ meaningful use criteria to reap financial benefits, I long ago abandoned that effort if earning benefits and reaching the desired patient outcome weren’t on the same path.
We are required to read numerous EHR windows, deal with misused, template-based medical records and the usually faulty copy-and-paste function, which results in flagrant errors. A common example is templating or copy-and-pasting normal examination findings such as “pupils equal, round, and reactive to light and accommodation (PERRLA),” without making modifications for a patient who has obvious abnormal pupillary findings. It is the EHR that often induces these types of documentation errors.
The EHR, as it exists now, intrudes into our time with patients. But for the past 30 years, the RBRVs have defined how we are compensated for our services. This compensation scale was created to provide a standard system of paying physicians’ services based on resource costs associated with patient care. The resource components are physician work, practice expense, and professional liability insurance. These components make our compensation based on effort rather than effect.
Payments are calculated into relative value units (RVUs), which are often structured into physician employment contracts.1 There are many RVU calculations and formulas that determine physician reimbursement and compensation; these are not entirely straightforward and too often lack transparency. Despite Dr Osler’s plea in Aequanimitas for physicians to maintain imperturbability and equanimity, that plea goes to the wayside when debating the value of the RBRVS. This system dilutes the complexities of the physician visit, especially for patients with comorbidities, polypharmacy, and cognitive and social concerns.2
Another frustrating, time-absorbing business requirement is E&M coding; the codes came about around the same time as RBRVs. Congress established E&M in the mid-1990s to facilitate medical billing by translating physician-patient encounters into 5-digit codes. In a neurology office, this authentication takes considerable effort, detracts from the patient’s visit, and adds to the documentation requirement to receive insured patient payments.
Years ago, I reviewed neurology insurance claims for a global health service company. I remember the considerable discussion over subjective documentation technicalities, attempting to justify the submitted E&M code. The onerous administrative burden E&M has created continues to evolve, with no end in sight.
Private insurance
When was the last time that you did not have to submit a prior authorization (PA) request to a payer in a week’s worth of days?
PA requests impede timely, efficient, and much-needed vital care while usurping a physician’s decision-making process. In 2020, the American Medical Association released the responses of 1000 physicians who were asked about making PA requests.3 Physicians said that the time delays affected their patients’ health and created adverse events, including hospitalizations. PAs are not only requested for new drugs; physicians report that the increase in the volume of PAs includes requests for existing drugs and services.
It takes staff days to make the requests; most medical practices interact with dozens of different health plans, all with different requirements related to PAs. Insurers often follow the lead of Medicare, and Medicare does not cover most self-injectable medications.4
I can report the same experiences. Ten years ago, private insurers rejected ~20% of my practice’s PA requests. Today, more than half of my patients need a PA from their insurer—often for 2 or 3 prescriptions each—and at least half of the requests are rejected. And, unlike 10 years ago, most of my requests are still denied after an appeal.
My patients are mostly migraine patients. When appropriate, I discuss with them the new acute and preventive anti–calcitonin gene-related peptides (anti-CGRPs), which, for the chronic migraine patient, can be a small slice of heaven. Reality strikes, however, when we discuss the likely PA process. This shift no longer focuses on getting likely migraine relief, but instead on the insurance company or companies approving the PA.
Sometimes the PA approval process is only accomplished by patients fighting the PA battle for themselves. One patient recently had to convince her PA oversight insurance representative that, if her PA was denied, her suicide would follow.
And what do patients do if the PA has been denied? Sometimes I must treat a patient with something else, which is often less appropriate for that patient. I have had many patients who have given up during the process.
Industry sees PAs in a different light. A survey5 of 44 payers conducted in 2019 found that PAs save money, improve evidence-based care, and so on. Physicians asking for the PAs were singled out as the reasons PAs were denied, as these physicians did not follow proper protocols.
Despite government and PHI policies that are supposed to enhance healthcare delivery and stabilize costs, US healthcare costs stand at $3.6 trillion.6,7 These medical practice transitions have increased administrative burden, accounting for 34% of US total healthcare expenditures vs 17% in Canada.8,9
In neurology, successful outcomes are predicated on recognizing the singularity of each patient. The current health system’s need for homogenization is making such recognition difficult. I invite you to read my commentary entitled The Practice of Medicine - Hazy or Invisible Lines, which discusses the unintended consequences of these well-intentioned medical practice adjustments.
Comments from Alan Rapoport, MD
Editor in Chief, Neurology Reviews
Professor Landy’s article excellently details just some of the roadblocks all neurologists face in providing patient-centric care. Prescribing medication or devices alone does not provide such care, but that is what many doctors must do because of limited time with the patient. Dr Osler was correct; we have to treat the patient who has the disease, not the disease the patient has. Taking an adequate history, conducting a full neurologic examination, documenting both, reviewing outside records, discussing the diagnosis and plan with the patient, ordering appropriate testing, and dictating all of the above in 20 or 30 minutes is impossible to do well. Going forward, we can expect computers and some form of artificial intelligence will help us to be more efficient, but we must keep the patient in the center. No wonder patients are not as happy with the healthcare system and their doctors as they used to be.
Alan Rapoport, MD
Clinical Professor of Neurology
The David Geffen School of Medicine at UCLA, Los Angeles, California
Past President
The International Headache Society (IHS)
References
- Menacker M. Physician compensation methodology must change! Am J Med. 2019;132(5):554-555.
2. Katz S, Melmed G. How relative value units undervalue the cognitive physician visit: a focus on inflammatory bowel disease. Gastroenterol Hepatol (N Y). 2016;12(4):240-244.
- American Medical Association. 2020 AMA prior authorization (PA) physician survey. 2020. Accessed March 10, 2022.
- Medicare. Prescription drugs (outpatient). Accessed March 9, 2022. https://www.medicare.gov/coverage/prescription-drugs-outpatient
- AHIP. Key Results of Industry Survey on Prior Authorizations. Published 2020. Accessed March 9, 2022. https://www.ahip.org/ahip-survey-prior-authorization-grounded-in-clinical-evidence-and-selectively-used/
- Agha L, Frandsen B, Rebitzer JB. Fragmented division of labor and healthcare costs: evidence from moves across regions. J Public Econ. 2019;169:144-159.
- Geng LN, Verghese A, Tilburt JC. Consultative medicine – an emerging specialty for patients with perplexing conditions. N Engl J Med. 2021;385(26):2478-2484.
- Kocher RP. Reducing administrative waste in the US health care system. JAMA. 2021;325(5):427-428.
- Himmelstein DU, Campbell T, Woolhandler S. Health care administrative costs in the United States and Canada, 2017. Ann Intern Med. 2020;172(2):134-142.
- Landy S. The practice of medicine – hazy or invisible lines. HPHR. Published 2021. Accessed March 9, 2022. hphr.org/blog-landy-1/
Neurology, once considered a “diagnose and adios” specialty, is gaining newfound, scientific respect. Our vastly improved understanding of neurologic pathophysiology has led to many Food and Drug Administration–approved medications that can specifically enhance treatment outcomes. Medications for migraine, multiple sclerosis, epilepsy, and other chronic neurological diseases have been extended and modernized; for millions of patients, these medicines fulfill their long-awaited needs.
What would Dr Osler have said if he witnessed today’s definition of the practice of medicine? As singular as our patients and their disorders are, the delivery of care is anything but. The processes in the delivery of this care have created many unforeseen twists and turns, thanks to the electronic health record (EHR), the resource-based relative value scale (RBRVS), evaluation and management (E&M) coding, and private health insurance (PHI).
From a neurologist’s perspective, I will elaborate upon these changes that have affected our day-to-day neurology practices. I have practiced general neurology and headache medicine both in private and academic practices, evaluating and treating thousands of inpatients and outpatients in urban and rural healthcare facilities since 1986.
The EHR
Despite herculean, lofty, and sustained efforts by the medical business world to promote EHR adoption worldwide, goals remain unmet. Intended to improve the quality of care and patient outcomes, reduce medical errors, and crystalize communications among providers and with patients, it is instead associated with physician burnout (B), lack of usability (U) and interoperability (I), has likability (L) issues, and provides no productive physician direction (D) – there is an enormous need to BUILD it better.
In my own practice, it is inevitable that I will use my EHR laptop with an unknowing patient. If so, I try to make her feel comfortable in its presence as I strive to stay intent on our discussion. Yet I invariably split my concentration between machine and patient. The machine often gets my full attention, with its confusing and unnecessary medical record notes, tech glitches and screen interruptions, let alone its complicated web of tabs, buttons, links, and obscure prompts. As for fulfilling CMS’ meaningful use criteria to reap financial benefits, I long ago abandoned that effort if earning benefits and reaching the desired patient outcome weren’t on the same path.
We are required to read numerous EHR windows, deal with misused, template-based medical records and the usually faulty copy-and-paste function, which results in flagrant errors. A common example is templating or copy-and-pasting normal examination findings such as “pupils equal, round, and reactive to light and accommodation (PERRLA),” without making modifications for a patient who has obvious abnormal pupillary findings. It is the EHR that often induces these types of documentation errors.
The EHR, as it exists now, intrudes into our time with patients. But for the past 30 years, the RBRVs have defined how we are compensated for our services. This compensation scale was created to provide a standard system of paying physicians’ services based on resource costs associated with patient care. The resource components are physician work, practice expense, and professional liability insurance. These components make our compensation based on effort rather than effect.
Payments are calculated into relative value units (RVUs), which are often structured into physician employment contracts.1 There are many RVU calculations and formulas that determine physician reimbursement and compensation; these are not entirely straightforward and too often lack transparency. Despite Dr Osler’s plea in Aequanimitas for physicians to maintain imperturbability and equanimity, that plea goes to the wayside when debating the value of the RBRVS. This system dilutes the complexities of the physician visit, especially for patients with comorbidities, polypharmacy, and cognitive and social concerns.2
Another frustrating, time-absorbing business requirement is E&M coding; the codes came about around the same time as RBRVs. Congress established E&M in the mid-1990s to facilitate medical billing by translating physician-patient encounters into 5-digit codes. In a neurology office, this authentication takes considerable effort, detracts from the patient’s visit, and adds to the documentation requirement to receive insured patient payments.
Years ago, I reviewed neurology insurance claims for a global health service company. I remember the considerable discussion over subjective documentation technicalities, attempting to justify the submitted E&M code. The onerous administrative burden E&M has created continues to evolve, with no end in sight.
Private insurance
When was the last time that you did not have to submit a prior authorization (PA) request to a payer in a week’s worth of days?
PA requests impede timely, efficient, and much-needed vital care while usurping a physician’s decision-making process. In 2020, the American Medical Association released the responses of 1000 physicians who were asked about making PA requests.3 Physicians said that the time delays affected their patients’ health and created adverse events, including hospitalizations. PAs are not only requested for new drugs; physicians report that the increase in the volume of PAs includes requests for existing drugs and services.
It takes staff days to make the requests; most medical practices interact with dozens of different health plans, all with different requirements related to PAs. Insurers often follow the lead of Medicare, and Medicare does not cover most self-injectable medications.4
I can report the same experiences. Ten years ago, private insurers rejected ~20% of my practice’s PA requests. Today, more than half of my patients need a PA from their insurer—often for 2 or 3 prescriptions each—and at least half of the requests are rejected. And, unlike 10 years ago, most of my requests are still denied after an appeal.
My patients are mostly migraine patients. When appropriate, I discuss with them the new acute and preventive anti–calcitonin gene-related peptides (anti-CGRPs), which, for the chronic migraine patient, can be a small slice of heaven. Reality strikes, however, when we discuss the likely PA process. This shift no longer focuses on getting likely migraine relief, but instead on the insurance company or companies approving the PA.
Sometimes the PA approval process is only accomplished by patients fighting the PA battle for themselves. One patient recently had to convince her PA oversight insurance representative that, if her PA was denied, her suicide would follow.
And what do patients do if the PA has been denied? Sometimes I must treat a patient with something else, which is often less appropriate for that patient. I have had many patients who have given up during the process.
Industry sees PAs in a different light. A survey5 of 44 payers conducted in 2019 found that PAs save money, improve evidence-based care, and so on. Physicians asking for the PAs were singled out as the reasons PAs were denied, as these physicians did not follow proper protocols.
Despite government and PHI policies that are supposed to enhance healthcare delivery and stabilize costs, US healthcare costs stand at $3.6 trillion.6,7 These medical practice transitions have increased administrative burden, accounting for 34% of US total healthcare expenditures vs 17% in Canada.8,9
In neurology, successful outcomes are predicated on recognizing the singularity of each patient. The current health system’s need for homogenization is making such recognition difficult. I invite you to read my commentary entitled The Practice of Medicine - Hazy or Invisible Lines, which discusses the unintended consequences of these well-intentioned medical practice adjustments.
Comments from Alan Rapoport, MD
Editor in Chief, Neurology Reviews
Professor Landy’s article excellently details just some of the roadblocks all neurologists face in providing patient-centric care. Prescribing medication or devices alone does not provide such care, but that is what many doctors must do because of limited time with the patient. Dr Osler was correct; we have to treat the patient who has the disease, not the disease the patient has. Taking an adequate history, conducting a full neurologic examination, documenting both, reviewing outside records, discussing the diagnosis and plan with the patient, ordering appropriate testing, and dictating all of the above in 20 or 30 minutes is impossible to do well. Going forward, we can expect computers and some form of artificial intelligence will help us to be more efficient, but we must keep the patient in the center. No wonder patients are not as happy with the healthcare system and their doctors as they used to be.
Alan Rapoport, MD
Clinical Professor of Neurology
The David Geffen School of Medicine at UCLA, Los Angeles, California
Past President
The International Headache Society (IHS)
Neurology, once considered a “diagnose and adios” specialty, is gaining newfound, scientific respect. Our vastly improved understanding of neurologic pathophysiology has led to many Food and Drug Administration–approved medications that can specifically enhance treatment outcomes. Medications for migraine, multiple sclerosis, epilepsy, and other chronic neurological diseases have been extended and modernized; for millions of patients, these medicines fulfill their long-awaited needs.
What would Dr Osler have said if he witnessed today’s definition of the practice of medicine? As singular as our patients and their disorders are, the delivery of care is anything but. The processes in the delivery of this care have created many unforeseen twists and turns, thanks to the electronic health record (EHR), the resource-based relative value scale (RBRVS), evaluation and management (E&M) coding, and private health insurance (PHI).
From a neurologist’s perspective, I will elaborate upon these changes that have affected our day-to-day neurology practices. I have practiced general neurology and headache medicine both in private and academic practices, evaluating and treating thousands of inpatients and outpatients in urban and rural healthcare facilities since 1986.
The EHR
Despite herculean, lofty, and sustained efforts by the medical business world to promote EHR adoption worldwide, goals remain unmet. Intended to improve the quality of care and patient outcomes, reduce medical errors, and crystalize communications among providers and with patients, it is instead associated with physician burnout (B), lack of usability (U) and interoperability (I), has likability (L) issues, and provides no productive physician direction (D) – there is an enormous need to BUILD it better.
In my own practice, it is inevitable that I will use my EHR laptop with an unknowing patient. If so, I try to make her feel comfortable in its presence as I strive to stay intent on our discussion. Yet I invariably split my concentration between machine and patient. The machine often gets my full attention, with its confusing and unnecessary medical record notes, tech glitches and screen interruptions, let alone its complicated web of tabs, buttons, links, and obscure prompts. As for fulfilling CMS’ meaningful use criteria to reap financial benefits, I long ago abandoned that effort if earning benefits and reaching the desired patient outcome weren’t on the same path.
We are required to read numerous EHR windows, deal with misused, template-based medical records and the usually faulty copy-and-paste function, which results in flagrant errors. A common example is templating or copy-and-pasting normal examination findings such as “pupils equal, round, and reactive to light and accommodation (PERRLA),” without making modifications for a patient who has obvious abnormal pupillary findings. It is the EHR that often induces these types of documentation errors.
The EHR, as it exists now, intrudes into our time with patients. But for the past 30 years, the RBRVs have defined how we are compensated for our services. This compensation scale was created to provide a standard system of paying physicians’ services based on resource costs associated with patient care. The resource components are physician work, practice expense, and professional liability insurance. These components make our compensation based on effort rather than effect.
Payments are calculated into relative value units (RVUs), which are often structured into physician employment contracts.1 There are many RVU calculations and formulas that determine physician reimbursement and compensation; these are not entirely straightforward and too often lack transparency. Despite Dr Osler’s plea in Aequanimitas for physicians to maintain imperturbability and equanimity, that plea goes to the wayside when debating the value of the RBRVS. This system dilutes the complexities of the physician visit, especially for patients with comorbidities, polypharmacy, and cognitive and social concerns.2
Another frustrating, time-absorbing business requirement is E&M coding; the codes came about around the same time as RBRVs. Congress established E&M in the mid-1990s to facilitate medical billing by translating physician-patient encounters into 5-digit codes. In a neurology office, this authentication takes considerable effort, detracts from the patient’s visit, and adds to the documentation requirement to receive insured patient payments.
Years ago, I reviewed neurology insurance claims for a global health service company. I remember the considerable discussion over subjective documentation technicalities, attempting to justify the submitted E&M code. The onerous administrative burden E&M has created continues to evolve, with no end in sight.
Private insurance
When was the last time that you did not have to submit a prior authorization (PA) request to a payer in a week’s worth of days?
PA requests impede timely, efficient, and much-needed vital care while usurping a physician’s decision-making process. In 2020, the American Medical Association released the responses of 1000 physicians who were asked about making PA requests.3 Physicians said that the time delays affected their patients’ health and created adverse events, including hospitalizations. PAs are not only requested for new drugs; physicians report that the increase in the volume of PAs includes requests for existing drugs and services.
It takes staff days to make the requests; most medical practices interact with dozens of different health plans, all with different requirements related to PAs. Insurers often follow the lead of Medicare, and Medicare does not cover most self-injectable medications.4
I can report the same experiences. Ten years ago, private insurers rejected ~20% of my practice’s PA requests. Today, more than half of my patients need a PA from their insurer—often for 2 or 3 prescriptions each—and at least half of the requests are rejected. And, unlike 10 years ago, most of my requests are still denied after an appeal.
My patients are mostly migraine patients. When appropriate, I discuss with them the new acute and preventive anti–calcitonin gene-related peptides (anti-CGRPs), which, for the chronic migraine patient, can be a small slice of heaven. Reality strikes, however, when we discuss the likely PA process. This shift no longer focuses on getting likely migraine relief, but instead on the insurance company or companies approving the PA.
Sometimes the PA approval process is only accomplished by patients fighting the PA battle for themselves. One patient recently had to convince her PA oversight insurance representative that, if her PA was denied, her suicide would follow.
And what do patients do if the PA has been denied? Sometimes I must treat a patient with something else, which is often less appropriate for that patient. I have had many patients who have given up during the process.
Industry sees PAs in a different light. A survey5 of 44 payers conducted in 2019 found that PAs save money, improve evidence-based care, and so on. Physicians asking for the PAs were singled out as the reasons PAs were denied, as these physicians did not follow proper protocols.
Despite government and PHI policies that are supposed to enhance healthcare delivery and stabilize costs, US healthcare costs stand at $3.6 trillion.6,7 These medical practice transitions have increased administrative burden, accounting for 34% of US total healthcare expenditures vs 17% in Canada.8,9
In neurology, successful outcomes are predicated on recognizing the singularity of each patient. The current health system’s need for homogenization is making such recognition difficult. I invite you to read my commentary entitled The Practice of Medicine - Hazy or Invisible Lines, which discusses the unintended consequences of these well-intentioned medical practice adjustments.
Comments from Alan Rapoport, MD
Editor in Chief, Neurology Reviews
Professor Landy’s article excellently details just some of the roadblocks all neurologists face in providing patient-centric care. Prescribing medication or devices alone does not provide such care, but that is what many doctors must do because of limited time with the patient. Dr Osler was correct; we have to treat the patient who has the disease, not the disease the patient has. Taking an adequate history, conducting a full neurologic examination, documenting both, reviewing outside records, discussing the diagnosis and plan with the patient, ordering appropriate testing, and dictating all of the above in 20 or 30 minutes is impossible to do well. Going forward, we can expect computers and some form of artificial intelligence will help us to be more efficient, but we must keep the patient in the center. No wonder patients are not as happy with the healthcare system and their doctors as they used to be.
Alan Rapoport, MD
Clinical Professor of Neurology
The David Geffen School of Medicine at UCLA, Los Angeles, California
Past President
The International Headache Society (IHS)
References
- Menacker M. Physician compensation methodology must change! Am J Med. 2019;132(5):554-555.
2. Katz S, Melmed G. How relative value units undervalue the cognitive physician visit: a focus on inflammatory bowel disease. Gastroenterol Hepatol (N Y). 2016;12(4):240-244.
- American Medical Association. 2020 AMA prior authorization (PA) physician survey. 2020. Accessed March 10, 2022.
- Medicare. Prescription drugs (outpatient). Accessed March 9, 2022. https://www.medicare.gov/coverage/prescription-drugs-outpatient
- AHIP. Key Results of Industry Survey on Prior Authorizations. Published 2020. Accessed March 9, 2022. https://www.ahip.org/ahip-survey-prior-authorization-grounded-in-clinical-evidence-and-selectively-used/
- Agha L, Frandsen B, Rebitzer JB. Fragmented division of labor and healthcare costs: evidence from moves across regions. J Public Econ. 2019;169:144-159.
- Geng LN, Verghese A, Tilburt JC. Consultative medicine – an emerging specialty for patients with perplexing conditions. N Engl J Med. 2021;385(26):2478-2484.
- Kocher RP. Reducing administrative waste in the US health care system. JAMA. 2021;325(5):427-428.
- Himmelstein DU, Campbell T, Woolhandler S. Health care administrative costs in the United States and Canada, 2017. Ann Intern Med. 2020;172(2):134-142.
- Landy S. The practice of medicine – hazy or invisible lines. HPHR. Published 2021. Accessed March 9, 2022. hphr.org/blog-landy-1/
References
- Menacker M. Physician compensation methodology must change! Am J Med. 2019;132(5):554-555.
2. Katz S, Melmed G. How relative value units undervalue the cognitive physician visit: a focus on inflammatory bowel disease. Gastroenterol Hepatol (N Y). 2016;12(4):240-244.
- American Medical Association. 2020 AMA prior authorization (PA) physician survey. 2020. Accessed March 10, 2022.
- Medicare. Prescription drugs (outpatient). Accessed March 9, 2022. https://www.medicare.gov/coverage/prescription-drugs-outpatient
- AHIP. Key Results of Industry Survey on Prior Authorizations. Published 2020. Accessed March 9, 2022. https://www.ahip.org/ahip-survey-prior-authorization-grounded-in-clinical-evidence-and-selectively-used/
- Agha L, Frandsen B, Rebitzer JB. Fragmented division of labor and healthcare costs: evidence from moves across regions. J Public Econ. 2019;169:144-159.
- Geng LN, Verghese A, Tilburt JC. Consultative medicine – an emerging specialty for patients with perplexing conditions. N Engl J Med. 2021;385(26):2478-2484.
- Kocher RP. Reducing administrative waste in the US health care system. JAMA. 2021;325(5):427-428.
- Himmelstein DU, Campbell T, Woolhandler S. Health care administrative costs in the United States and Canada, 2017. Ann Intern Med. 2020;172(2):134-142.
- Landy S. The practice of medicine – hazy or invisible lines. HPHR. Published 2021. Accessed March 9, 2022. hphr.org/blog-landy-1/
Cough and moderate hoarseness
Based on the patient's presentation, history, and imaging results, the likely diagnosis is non–small cell lung cancer (NSCLC) of an adenocarcinoma subtype. NSCLC accounts for about 80% of all lung cancer cases. Adenocarcinoma, in particular, is the most common type of lung cancer in the United States, accounting for about 40% of cases. This subtype is also the most common histology among nonsmokers. Still, individuals aged 55 to 77 years with a smoking history of 30 pack-years or more are considered to be the highest-risk group for lung cancer; those who quit less than 15 years ago — like the patient in the present case — are still considered to be in this risk group. Most cases of lung cancer are diagnosed at a late stage when symptoms have already begun to manifest. However, it should be noted that women are more likely to develop adenocarcinoma, are generally younger when they present with symptoms, and are more likely to present with localized disease. It remains to be proven whether the use of HRT affects the risk for lung cancer in women. Deaths from lung cancer, and in particular NSCLC, were shown to be higher among patients undergoing HRT, though no increase in lung cancer death was reported in women receiving estrogen alone.
In addition to the imaging described in this case, workup for NSCLC should include immunohistochemical (IHC) analyses to identify tumor type and lineage (adenocarcinoma, squamous cell carcinoma, metastatic malignancy, or primary pleural mesothelioma). Separate IHC analyses are then used to guide treatment decisions, identifying whether anaplastic lymphoma kinase inhibitor therapy or programmed death-ligand 1 inhibitor therapy would be appropriate.
Tissue should also be conserved for molecular testing. Management of NSCLC is primarily informed by the presence of targetable mutations. Among adenocarcinoma cases, the most common mutations are in the EGFR and KRAS genes. KRAS mutations, unlike EGFR mutations, are associated with a history of smoking and are considered prognostic biomarkers. Because overlapping targetable alterations are uncommon, patients who are confirmed to be harboring KRAS mutations will likely not benefit from additional molecular testing. Presence of the KRAS mutation suggests a poor response to EGFR tyrosine kinase inhibitors, though it does not appear to impact chemotherapeutic efficacy. Although no targeted therapies are yet available for this population, immune checkpoint inhibitors appear to be beneficial. National Comprehensive Cancer Network guidelines advise that all patients with adenocarcinoma be tested for EGFR mutations and that DNA mutational analysis is the preferred method.
Karl J. D'Silva, MD, Clinical Assistant Professor, Department of Medicine, Tufts University School of Medicine, Boston; Medical Director, Department of Oncology and Hematology, Lahey Hospital and Medical Center, Peabody, Massachusetts.
Karl J. D'Silva, MD, has disclosed no relevant financial relationships.
Based on the patient's presentation, history, and imaging results, the likely diagnosis is non–small cell lung cancer (NSCLC) of an adenocarcinoma subtype. NSCLC accounts for about 80% of all lung cancer cases. Adenocarcinoma, in particular, is the most common type of lung cancer in the United States, accounting for about 40% of cases. This subtype is also the most common histology among nonsmokers. Still, individuals aged 55 to 77 years with a smoking history of 30 pack-years or more are considered to be the highest-risk group for lung cancer; those who quit less than 15 years ago — like the patient in the present case — are still considered to be in this risk group. Most cases of lung cancer are diagnosed at a late stage when symptoms have already begun to manifest. However, it should be noted that women are more likely to develop adenocarcinoma, are generally younger when they present with symptoms, and are more likely to present with localized disease. It remains to be proven whether the use of HRT affects the risk for lung cancer in women. Deaths from lung cancer, and in particular NSCLC, were shown to be higher among patients undergoing HRT, though no increase in lung cancer death was reported in women receiving estrogen alone.
In addition to the imaging described in this case, workup for NSCLC should include immunohistochemical (IHC) analyses to identify tumor type and lineage (adenocarcinoma, squamous cell carcinoma, metastatic malignancy, or primary pleural mesothelioma). Separate IHC analyses are then used to guide treatment decisions, identifying whether anaplastic lymphoma kinase inhibitor therapy or programmed death-ligand 1 inhibitor therapy would be appropriate.
Tissue should also be conserved for molecular testing. Management of NSCLC is primarily informed by the presence of targetable mutations. Among adenocarcinoma cases, the most common mutations are in the EGFR and KRAS genes. KRAS mutations, unlike EGFR mutations, are associated with a history of smoking and are considered prognostic biomarkers. Because overlapping targetable alterations are uncommon, patients who are confirmed to be harboring KRAS mutations will likely not benefit from additional molecular testing. Presence of the KRAS mutation suggests a poor response to EGFR tyrosine kinase inhibitors, though it does not appear to impact chemotherapeutic efficacy. Although no targeted therapies are yet available for this population, immune checkpoint inhibitors appear to be beneficial. National Comprehensive Cancer Network guidelines advise that all patients with adenocarcinoma be tested for EGFR mutations and that DNA mutational analysis is the preferred method.
Karl J. D'Silva, MD, Clinical Assistant Professor, Department of Medicine, Tufts University School of Medicine, Boston; Medical Director, Department of Oncology and Hematology, Lahey Hospital and Medical Center, Peabody, Massachusetts.
Karl J. D'Silva, MD, has disclosed no relevant financial relationships.
Based on the patient's presentation, history, and imaging results, the likely diagnosis is non–small cell lung cancer (NSCLC) of an adenocarcinoma subtype. NSCLC accounts for about 80% of all lung cancer cases. Adenocarcinoma, in particular, is the most common type of lung cancer in the United States, accounting for about 40% of cases. This subtype is also the most common histology among nonsmokers. Still, individuals aged 55 to 77 years with a smoking history of 30 pack-years or more are considered to be the highest-risk group for lung cancer; those who quit less than 15 years ago — like the patient in the present case — are still considered to be in this risk group. Most cases of lung cancer are diagnosed at a late stage when symptoms have already begun to manifest. However, it should be noted that women are more likely to develop adenocarcinoma, are generally younger when they present with symptoms, and are more likely to present with localized disease. It remains to be proven whether the use of HRT affects the risk for lung cancer in women. Deaths from lung cancer, and in particular NSCLC, were shown to be higher among patients undergoing HRT, though no increase in lung cancer death was reported in women receiving estrogen alone.
In addition to the imaging described in this case, workup for NSCLC should include immunohistochemical (IHC) analyses to identify tumor type and lineage (adenocarcinoma, squamous cell carcinoma, metastatic malignancy, or primary pleural mesothelioma). Separate IHC analyses are then used to guide treatment decisions, identifying whether anaplastic lymphoma kinase inhibitor therapy or programmed death-ligand 1 inhibitor therapy would be appropriate.
Tissue should also be conserved for molecular testing. Management of NSCLC is primarily informed by the presence of targetable mutations. Among adenocarcinoma cases, the most common mutations are in the EGFR and KRAS genes. KRAS mutations, unlike EGFR mutations, are associated with a history of smoking and are considered prognostic biomarkers. Because overlapping targetable alterations are uncommon, patients who are confirmed to be harboring KRAS mutations will likely not benefit from additional molecular testing. Presence of the KRAS mutation suggests a poor response to EGFR tyrosine kinase inhibitors, though it does not appear to impact chemotherapeutic efficacy. Although no targeted therapies are yet available for this population, immune checkpoint inhibitors appear to be beneficial. National Comprehensive Cancer Network guidelines advise that all patients with adenocarcinoma be tested for EGFR mutations and that DNA mutational analysis is the preferred method.
Karl J. D'Silva, MD, Clinical Assistant Professor, Department of Medicine, Tufts University School of Medicine, Boston; Medical Director, Department of Oncology and Hematology, Lahey Hospital and Medical Center, Peabody, Massachusetts.
Karl J. D'Silva, MD, has disclosed no relevant financial relationships.
A 56-year-old woman presents with dyspnea, a persistent cough, and moderate hoarseness. She has no significant medical history other than thyroiditis. Her current medications include hormone replacement therapy (HRT). Although the patient reports a 20–pack-year history of smoking tobacco, she notes that she quit 11 years ago and has not been previously screened for lung cancer. A chest radiograph is ordered, which demonstrates a mass in the upper lobe of the right lung.
Type 2 Diabetes Comorbidities
Migraine Pathophysiology
Infectious disease pop quiz: Clinical challenge #18 for the ObGyn
What antenatal treatment is indicated in a pregnant woman at 28 weeks’ gestation who has a hepatitis B viral load of 2 million copies/mL?
Continue to the answer...
This patient has a markedly elevated viral load and is at significantly increased risk of transmitting hepatitis B infection to her neonate even if the infant receives hepatitis B immune globulin immediately after birth and quickly begins the hepatitis B vaccine series. Daily antenatal treatment with tenofovir (300 mg daily) from 28 weeks until delivery will significantly reduce the risk of perinatal transmission.
- Duff P. Maternal and perinatal infections: bacterial. In: Landon MB, Galan HL, Jauniaux ERM, et al. Gabbe’s Obstetrics: Normal and Problem Pregnancies. 8th ed. Elsevier; 2021:1124-1146.
- Duff P. Maternal and fetal infections. In: Resnik R, Lockwood CJ, Moore TJ, et al. Creasy & Resnik’s Maternal-Fetal Medicine: Principles and Practice. 8th ed. Elsevier; 2019:862-919.
What antenatal treatment is indicated in a pregnant woman at 28 weeks’ gestation who has a hepatitis B viral load of 2 million copies/mL?
Continue to the answer...
This patient has a markedly elevated viral load and is at significantly increased risk of transmitting hepatitis B infection to her neonate even if the infant receives hepatitis B immune globulin immediately after birth and quickly begins the hepatitis B vaccine series. Daily antenatal treatment with tenofovir (300 mg daily) from 28 weeks until delivery will significantly reduce the risk of perinatal transmission.
What antenatal treatment is indicated in a pregnant woman at 28 weeks’ gestation who has a hepatitis B viral load of 2 million copies/mL?
Continue to the answer...
This patient has a markedly elevated viral load and is at significantly increased risk of transmitting hepatitis B infection to her neonate even if the infant receives hepatitis B immune globulin immediately after birth and quickly begins the hepatitis B vaccine series. Daily antenatal treatment with tenofovir (300 mg daily) from 28 weeks until delivery will significantly reduce the risk of perinatal transmission.
- Duff P. Maternal and perinatal infections: bacterial. In: Landon MB, Galan HL, Jauniaux ERM, et al. Gabbe’s Obstetrics: Normal and Problem Pregnancies. 8th ed. Elsevier; 2021:1124-1146.
- Duff P. Maternal and fetal infections. In: Resnik R, Lockwood CJ, Moore TJ, et al. Creasy & Resnik’s Maternal-Fetal Medicine: Principles and Practice. 8th ed. Elsevier; 2019:862-919.
- Duff P. Maternal and perinatal infections: bacterial. In: Landon MB, Galan HL, Jauniaux ERM, et al. Gabbe’s Obstetrics: Normal and Problem Pregnancies. 8th ed. Elsevier; 2021:1124-1146.
- Duff P. Maternal and fetal infections. In: Resnik R, Lockwood CJ, Moore TJ, et al. Creasy & Resnik’s Maternal-Fetal Medicine: Principles and Practice. 8th ed. Elsevier; 2019:862-919.
Labial growth
White-to-pink friable plaques occurring acutely in the vulva is concerning for one form of secondary syphilis that affects mucous membranes: condyloma lata.
Known as the great imitator for its variety of clinical presentations, syphilis is a sexually transmitted infection (STI) caused by the spirochete Treponema pallidum. Three to 10 days following contact with the spirochete, a painless ulcer or chancre forms and subsequently resolves—sometimes without notice.
Secondary syphilis develops from hematogenous spread of bacteria taking many forms—most commonly a widespread rash over the whole body of many (although sometimes faint) macules or papules up to about 1 cm in size and haphazardly spread out about every 1 cm. Palms and soles may be affected, even if faintly. Another, less common form of secondary syphilis includes the friable plaques (often in the anogenital area, as pictured) that are highly concentrated with bacteria. These occur 3 to 12 weeks after the appearance of a primary chancre and are variably symptomatic.
The differential diagnosis includes genital warts, vulvar carcinoma, and pemphigus vegetans. The relatively rapid, multifocal presentation helps to separate this disorder from vulvar carcinoma. A biopsy can distinguish the 2. However, diagnosis is better made with serology using nontreponemal tests, such as the rapid plasma reagin (RPR) test. Treponemal tests (assaying immunoglobulin [Ig]M and IgG to Treponema pallidum) are also an option and are very specific. Following this, an RPR titer can help guide treatment. Darkfield microscopy, which can reveal spirochetes directly, isn’t readily available but could be used to diagnose condyloma lata.
Patients who have been given a diagnosis of syphilis should be offered screening for other STIs, including HIV. Anyone who has had sexual contact with the patient within the previous 90 days should be notified, tested, and treated. Patients with primary or secondary syphilis should be treated with 2.4 million units of intramuscular (IM) benzathine penicillin G in a single dose—regardless of whether they test positive for HIV. To exclude tertiary syphilis, a careful neurologic exam should take place at the time of diagnosis and again 6 and 12 months after treatment (sooner if follow-up may be uncertain). Consider treatment failure if RPR titers haven’t fallen fourfold in 12 months. In 2022, the Centers for Disease Control and Prevention released a notice that COVID-19-vaccinated patients may have false-positive RPR titers performed from Bio-Rad Laboratories (BioPlex 2200 Syphilis Total & RPR kit).1
In this case, the patient tested positive for treponemal antibodies and had an RPR titer of 1:128. She was treated with IM benzathine penicillin with lasting clearance.
Text courtesy of Jonathan Karnes, MD, medical director, MDFMR Dermatology Services, Augusta, ME. Photos courtesy of Jonathan Karnes, MD (copyright retained).
1. Centers for Disease Control and Prevention. Sexually Transmitted Infection Treatment Guidelines, 2021. Reviewed December 22, 2021. Accessed February 25, 2022. www.cdc.gov/std/treatment-guidelines/syphilis.htm
White-to-pink friable plaques occurring acutely in the vulva is concerning for one form of secondary syphilis that affects mucous membranes: condyloma lata.
Known as the great imitator for its variety of clinical presentations, syphilis is a sexually transmitted infection (STI) caused by the spirochete Treponema pallidum. Three to 10 days following contact with the spirochete, a painless ulcer or chancre forms and subsequently resolves—sometimes without notice.
Secondary syphilis develops from hematogenous spread of bacteria taking many forms—most commonly a widespread rash over the whole body of many (although sometimes faint) macules or papules up to about 1 cm in size and haphazardly spread out about every 1 cm. Palms and soles may be affected, even if faintly. Another, less common form of secondary syphilis includes the friable plaques (often in the anogenital area, as pictured) that are highly concentrated with bacteria. These occur 3 to 12 weeks after the appearance of a primary chancre and are variably symptomatic.
The differential diagnosis includes genital warts, vulvar carcinoma, and pemphigus vegetans. The relatively rapid, multifocal presentation helps to separate this disorder from vulvar carcinoma. A biopsy can distinguish the 2. However, diagnosis is better made with serology using nontreponemal tests, such as the rapid plasma reagin (RPR) test. Treponemal tests (assaying immunoglobulin [Ig]M and IgG to Treponema pallidum) are also an option and are very specific. Following this, an RPR titer can help guide treatment. Darkfield microscopy, which can reveal spirochetes directly, isn’t readily available but could be used to diagnose condyloma lata.
Patients who have been given a diagnosis of syphilis should be offered screening for other STIs, including HIV. Anyone who has had sexual contact with the patient within the previous 90 days should be notified, tested, and treated. Patients with primary or secondary syphilis should be treated with 2.4 million units of intramuscular (IM) benzathine penicillin G in a single dose—regardless of whether they test positive for HIV. To exclude tertiary syphilis, a careful neurologic exam should take place at the time of diagnosis and again 6 and 12 months after treatment (sooner if follow-up may be uncertain). Consider treatment failure if RPR titers haven’t fallen fourfold in 12 months. In 2022, the Centers for Disease Control and Prevention released a notice that COVID-19-vaccinated patients may have false-positive RPR titers performed from Bio-Rad Laboratories (BioPlex 2200 Syphilis Total & RPR kit).1
In this case, the patient tested positive for treponemal antibodies and had an RPR titer of 1:128. She was treated with IM benzathine penicillin with lasting clearance.
Text courtesy of Jonathan Karnes, MD, medical director, MDFMR Dermatology Services, Augusta, ME. Photos courtesy of Jonathan Karnes, MD (copyright retained).
White-to-pink friable plaques occurring acutely in the vulva is concerning for one form of secondary syphilis that affects mucous membranes: condyloma lata.
Known as the great imitator for its variety of clinical presentations, syphilis is a sexually transmitted infection (STI) caused by the spirochete Treponema pallidum. Three to 10 days following contact with the spirochete, a painless ulcer or chancre forms and subsequently resolves—sometimes without notice.
Secondary syphilis develops from hematogenous spread of bacteria taking many forms—most commonly a widespread rash over the whole body of many (although sometimes faint) macules or papules up to about 1 cm in size and haphazardly spread out about every 1 cm. Palms and soles may be affected, even if faintly. Another, less common form of secondary syphilis includes the friable plaques (often in the anogenital area, as pictured) that are highly concentrated with bacteria. These occur 3 to 12 weeks after the appearance of a primary chancre and are variably symptomatic.
The differential diagnosis includes genital warts, vulvar carcinoma, and pemphigus vegetans. The relatively rapid, multifocal presentation helps to separate this disorder from vulvar carcinoma. A biopsy can distinguish the 2. However, diagnosis is better made with serology using nontreponemal tests, such as the rapid plasma reagin (RPR) test. Treponemal tests (assaying immunoglobulin [Ig]M and IgG to Treponema pallidum) are also an option and are very specific. Following this, an RPR titer can help guide treatment. Darkfield microscopy, which can reveal spirochetes directly, isn’t readily available but could be used to diagnose condyloma lata.
Patients who have been given a diagnosis of syphilis should be offered screening for other STIs, including HIV. Anyone who has had sexual contact with the patient within the previous 90 days should be notified, tested, and treated. Patients with primary or secondary syphilis should be treated with 2.4 million units of intramuscular (IM) benzathine penicillin G in a single dose—regardless of whether they test positive for HIV. To exclude tertiary syphilis, a careful neurologic exam should take place at the time of diagnosis and again 6 and 12 months after treatment (sooner if follow-up may be uncertain). Consider treatment failure if RPR titers haven’t fallen fourfold in 12 months. In 2022, the Centers for Disease Control and Prevention released a notice that COVID-19-vaccinated patients may have false-positive RPR titers performed from Bio-Rad Laboratories (BioPlex 2200 Syphilis Total & RPR kit).1
In this case, the patient tested positive for treponemal antibodies and had an RPR titer of 1:128. She was treated with IM benzathine penicillin with lasting clearance.
Text courtesy of Jonathan Karnes, MD, medical director, MDFMR Dermatology Services, Augusta, ME. Photos courtesy of Jonathan Karnes, MD (copyright retained).
1. Centers for Disease Control and Prevention. Sexually Transmitted Infection Treatment Guidelines, 2021. Reviewed December 22, 2021. Accessed February 25, 2022. www.cdc.gov/std/treatment-guidelines/syphilis.htm
1. Centers for Disease Control and Prevention. Sexually Transmitted Infection Treatment Guidelines, 2021. Reviewed December 22, 2021. Accessed February 25, 2022. www.cdc.gov/std/treatment-guidelines/syphilis.htm
Painful swelling of fingers and toe
Although psoriatic arthritis is not the only disease associated with dactylitis — other culprits are sarcoidosis, septic arthritis, tuberculosis, and gout — dactylitis is one of the characteristic symptoms of psoriatic arthritis. Dactylitis is seen in as many as 35% of patients with psoriatic disease. Dactylitis clinically presents — as in this patient — with sausage-like swelling of the digits. It is included in the Classification Criteria for Psoriatic Arthritis (CASPAR) as one of the hallmarks of psoriatic arthritis.
Dactylitis has been thought to be a result of the concomitant swelling and inflammation of the flexor tendon sheaths of the metacarpophalangeal, metatarsophalangeal, or interphalangeal joints. Flexor tenosynovitis can be detected by examination with MRI and ultrasound. Dactylitis is associated with radiologically evident erosive damage to the joints.
Patients with psoriatic arthritis are typically seronegative for rheumatoid factor and antinuclear antibody; antinuclear antibody titers in persons with psoriatic arthritis do not differ from those of age- and sex-matched controls. C-reactive protein may be elevated but is often normal. Lack of C-reactive protein elevation, however, does not mean that systemic inflammation is absent, but rather indicates that different markers are needed that allow better quantification of systemic inflammation in psoriasis and psoriatic arthritis.
Herbert S. Diamond, MD, Professor of Medicine (retired), Temple University School of Medicine, University of Pittsburgh; Chairman, Department of Medicine Emeritus, Western Pennsylvania Hospital, Pittsburgh, PA.
Herbert S. Diamond, MD, has disclosed no relevant financial relationships.
Although psoriatic arthritis is not the only disease associated with dactylitis — other culprits are sarcoidosis, septic arthritis, tuberculosis, and gout — dactylitis is one of the characteristic symptoms of psoriatic arthritis. Dactylitis is seen in as many as 35% of patients with psoriatic disease. Dactylitis clinically presents — as in this patient — with sausage-like swelling of the digits. It is included in the Classification Criteria for Psoriatic Arthritis (CASPAR) as one of the hallmarks of psoriatic arthritis.
Dactylitis has been thought to be a result of the concomitant swelling and inflammation of the flexor tendon sheaths of the metacarpophalangeal, metatarsophalangeal, or interphalangeal joints. Flexor tenosynovitis can be detected by examination with MRI and ultrasound. Dactylitis is associated with radiologically evident erosive damage to the joints.
Patients with psoriatic arthritis are typically seronegative for rheumatoid factor and antinuclear antibody; antinuclear antibody titers in persons with psoriatic arthritis do not differ from those of age- and sex-matched controls. C-reactive protein may be elevated but is often normal. Lack of C-reactive protein elevation, however, does not mean that systemic inflammation is absent, but rather indicates that different markers are needed that allow better quantification of systemic inflammation in psoriasis and psoriatic arthritis.
Herbert S. Diamond, MD, Professor of Medicine (retired), Temple University School of Medicine, University of Pittsburgh; Chairman, Department of Medicine Emeritus, Western Pennsylvania Hospital, Pittsburgh, PA.
Herbert S. Diamond, MD, has disclosed no relevant financial relationships.
Although psoriatic arthritis is not the only disease associated with dactylitis — other culprits are sarcoidosis, septic arthritis, tuberculosis, and gout — dactylitis is one of the characteristic symptoms of psoriatic arthritis. Dactylitis is seen in as many as 35% of patients with psoriatic disease. Dactylitis clinically presents — as in this patient — with sausage-like swelling of the digits. It is included in the Classification Criteria for Psoriatic Arthritis (CASPAR) as one of the hallmarks of psoriatic arthritis.
Dactylitis has been thought to be a result of the concomitant swelling and inflammation of the flexor tendon sheaths of the metacarpophalangeal, metatarsophalangeal, or interphalangeal joints. Flexor tenosynovitis can be detected by examination with MRI and ultrasound. Dactylitis is associated with radiologically evident erosive damage to the joints.
Patients with psoriatic arthritis are typically seronegative for rheumatoid factor and antinuclear antibody; antinuclear antibody titers in persons with psoriatic arthritis do not differ from those of age- and sex-matched controls. C-reactive protein may be elevated but is often normal. Lack of C-reactive protein elevation, however, does not mean that systemic inflammation is absent, but rather indicates that different markers are needed that allow better quantification of systemic inflammation in psoriasis and psoriatic arthritis.
Herbert S. Diamond, MD, Professor of Medicine (retired), Temple University School of Medicine, University of Pittsburgh; Chairman, Department of Medicine Emeritus, Western Pennsylvania Hospital, Pittsburgh, PA.
Herbert S. Diamond, MD, has disclosed no relevant financial relationships.
A 35-year-old man presents with painful swelling of his right index and ring fingers as well as the fourth toe on his right foot, which has persisted for 5 days. He cannot perform his daily activities owing to severe pain in the affected fingers and toes. His medical history is unremarkable. His paternal uncle had psoriasis, which was successfully treated with adalimumab.
Physical assessment reveals tender, fusiform, swollen soft tissues in the affected fingertips, the fourth toe, and swollen palms. Nails are pitted. Hand radiography reveals mild edema of the soft tissue of the index and ring fingers but no significant joint abnormalities. Enthesitis is not present. Laboratory tests reveal a negative human leukocyte antigen B27 (HLA-B27) test, negative rheumatoid factor, negative antinuclear antibody, and normal C-reactive protein.
Dactylitis was diagnosed on the basis of clinical symptoms, radiographic results, and laboratory findings.










